ChaseDream
搜索
返回列表 发新帖
查看: 78340|回复: 70
打印 上一主题 下一主题

GWD13逻辑详解(我的复习笔记,题目后面有参考思路)

[精华] [复制链接]
楼主
发表于 2007-5-17 15:10:00 | 只看该作者

GWD13逻辑详解(我的复习笔记,题目后面有参考思路)

说明:

        这是偶备考时的逻辑复习笔记,答案大家应该都有,但偶觉得这个笔记如果还有点价值的话,就是基本每个题后面都有这个题的参考思路(也参照了前人NN们的一些思路和讨论,个别很简单的就没有写),还有些就是我做错的题,也在后面注明了当时我为什么会错,是个别词没看懂,属于理解的问题,如果是理解有误的话,那错误思路是什么,正确的思路是什么——我觉得思路是最重要的,尤其对逻辑题来讲。

        这些思路和理解中,有些也不完全正确(就象GWD,本来也没有一个确实的“标准答案”)希望大家有批判地看,并在最终复习中形成自己一套行之有效的解题思路,注:有些前后重复的题号也标注上了。

祝各位好运。

      

GWD 13CR分类总结

 

 ------------------------------------------------------------------------------------------------------------

GWD-1-Q3:

A product that represents a clear technological advance over competing products can generally command a high price.  Because technological advances tend to be quickly surpassed and companies want to make large profits while they still can, many companies charge the greatest price the market will bear when they have such a product.  But
    large profits on the mew product will give competitors a strong incentive to quickly match the mew product’s capabilities.  Consequently, the strategy to maximize overall profit from a new product is to charge less than the greatest possible price.

In the argument above, the two portions in boldface play which of the following roles?

  1. The first is an assumption that forms the basis for a course of action that the argument criticizes; the second presents the course of action endorsed by the argument.

  2. The first is a consideration raised to explain the appeal of a certain strategy; the second is a consideration raised to call into question the wisdom of adopting that strategy.

  3. The first is an assumption that has been used to justify a certain strategy; the second is a consideration that is used to cast doubt on that assumption.

  4. The first is a consideration raised in support of a strategy the argument endorses; the second presents grounds in support of that consideration.

  5. The first is a consideration raised to show that adopting a certain strategy is unlikely to achieve the intended effect; the second is presented to explain the appeal of that strategy.

第一步不是假设,AC去掉。有转折,DE去掉。

(B)

------------------------------------------------------------------------------------------------------------

GWD-1-Q13:

In the United States, of the people who moved from one state to another when they retired, the percentage who retired to Florida has decreased by three percentage points over the past ten years.  Since many local businesses in Florida cater to retirees, this decline is likely to have a noticeably negative economic effect on these businesses.

Which of the following, if true, most seriously weakens the argument?

  1. Florida attracts more people who move from one state to another when they retire than does any other state.

  2. The number of people who move out of Florida to accept employment in other states has increased over the past ten years.

  3. There are far more local businesses in Florida that cater to tourists than there are local businesses that cater to retirees.

  4. The total number of people who retired and moved to another state for their retirement has increased significantly over the past ten years.

  5. The number of people who left Florida when they retired to live in another state was greater last year than it was ten years ago.

[D]

错选为A   A是无关,D的意思是,即使比例下降了,但绝对数上升了,所以这些生意还是好的。而且是和佛州本身比,不是和其它州比,A错。本题考察numbert and percentageGMAT的一大类题。

------------------------------------------------------------------------------------------------------------

GWD-1-Q14:

That the application of new technology can increase the productivity of existing coal mines is demonstrated by the case of Tribnia’s coal industry.  Coal output per miner in Tribnia is double what it was five years ago, even though no new mines have opened.

 Which of the following can be properly concluded from the statement about coal output per miner in the passage? 

 Which of the following can be properly concluded from the statement about coal output per miner in the passage? 

 Which of the following can be properly concluded from the statement about coal output per miner in the passage? 

  1. If the number of miners working in Tribnian coal mines has remained constant in the past five years, Tribnia’s total coal production has doubled in that period of time.

  2. Any individual Tribnian coal mine that achieved an increase in overall output in the past five years has also experienced an increase in output per miner.

  3. If any new coal mines had opened in Tribnia in the past five years, then the increase in output per miner would have been even greater than it actually was.

  4. If any individual Tribnian coal mine has not increased its output per miner in the past five years, then that mine’s overall output has declined or remained constant.

  5. In Tribnia the cost of producing a given quantity of coal has declined over the past five years.

又是一个数学题每名矿工的生产率在5年时间内翻了一倍,所以A说如果矿工数量不变,那么总产量肯定是翻了一倍了。

总产量=矿工数量x每名矿工的生产率

 [A]

------------------------------------------------------------------------------------------------------------

GWD-1-Q15:

In parts of South America, vitamin-A deficiency is a serious health problem, especially among children.  In one region, agriculturists are attempting to improve nutrition by encouraging farmers to plant a new variety of sweet potato called SPK004 that is rich in beta-carotene, which the body converts into vitamin A.  The plan has good chances of success, since sweet potato is a staple主要成份 of the region’s diet and agriculture, and the varieties currently grown contain little beta-carotene.

Which of the following, if true, most strongly supports the prediction that the plan will succeed? 

Which of the following, if true, most strongly supports the prediction that the plan will succeed? 

Which of the following, if true, most strongly supports the prediction that the plan will succeed? 

 

  1. The growing conditions required by the varieties of sweet potato currently cultivated in the region are conditions in which SPK004 can flourish.

  2. The flesh of SPK004 differs from that of the currently cultivated sweet potatoes in color and texture, so traditional foods would look somewhat different when prepared from SPK004.

  3. There are no other varieties of sweet potato that are significantly richer in beta-carotene than SPK004 is.

  4. The varieties of sweet potato currently cultivated in the region contain some important nutrients that are lacking in SPK004.

  5. There are other vegetables currently grown in the region that contain more beta-carotene than the currently cultivated varieties of sweet potato do.

[A]

------------------------------------------------------------------------------------------------------------

GWD-1-Q17:

The spacing of the four holes on a fragment of a bone flute excavated at a Neanderthal campsite is just what is required to play the third through sixth notes of the diatonic scale—the seven-note musical scale used in much of Western music since the Renaissance.  Musicologists therefore hypothesize that the diatonic musical scale was developed and used thousands of years before it was adopted by Western musicians.

 Which of the following, if true, most strongly supports the hypothesis?

 Which of the following, if true, most strongly supports the hypothesis?

 Which of the following, if true, most strongly supports the hypothesis?

 

 

 

  1. Bone flutes were probably the only musical instrument made by Neanderthals.

  2. No musical instrument that is known to have used a diatomic scale is of an earlier date than the flute found at the Neanderthal campsite.

  3. The flute was made from a cave-bear bone and the campsite at which the flute fragment was excavated was in a cave that also contained skeletal remains of cave bears.

  4. Flutes are the simplest wind instrument that can be constructed to allow playing a diatonic scale.

  5. The cave-bear leg bone used to make the Neanderthal flute would have been long enough to make a flute capable of playing a complete diatonic scale.

这个笛子只有四个孔残留,而结论认为这个四孔的骨笛就表明了当时就是7音音阶,则实际上是假定这个骨笛当时应该是至少有7孔的,e支持了这个假设。

------------------------------------------------------------------------------------------------------------

GWD-1-Q18:

It is illegal to advertise prescription medications in Hedland except in professional medical journals or by mail directly to physicians.  A proposed law would allow general advertising of prescription medications.  Opponents object that, in general, laypersons lack the specialized knowledge to evaluate such advertisements and might ask their physicians for inappropriate medications.  But since physicians have the final say as to whether to prescribe a medication for a patient, the objection provides no grounds for concern.

 Which of the following would it be most useful to establish in order to evaluate the argument?

 Which of the following would it be most useful to establish in order to evaluate the argument?

 Which of the following would it be most useful to establish in order to evaluate the argument?

  

  

  

  1. Whether nonprescription medications can interact with and block the action of any prescription medications that could be advertised to the general public

  2. Whether most prescription medication advertisements directed at the general public would be advertisements for recently developed medications newly available by prescription

  3. Whether prescription medication advertisements directed at the general public would appear on television and radio as well as in print

  4. Whether physicians are more likely to pay attention to advertising directed to the general public than to advertising directed to physicians

  5. Whether physicians are likely to succumb to pressure from patients to prescribe inappropriate medications [E]

A与非处方药无关,Bnewly availableC的广播电视广告无关。D医生是否更注意直接给医生的广告还是登在公众出版物上无关。题目中含有患者找医生开药,选项中应该有“医生是否会给开”这样的意思。

------------------------------------------------------------------------------------------------------------

GWD-1-Q22:
      

GWD-8-Q31:

City Official:  At City
     Hospital, uninsured patients tend to have shorter stays and fewer procedures performed than do insured patients, even though insured patients, on average, have slightly less serious medical problems at the time of admission to the hospital than uninsured patients have.  Critics of the hospital have concluded that the uninsured patients are
not receiving proper medical care.  However, this conclusion is almost certainly false.  Careful investigation has recently shown two things:  insured patients have much longer stays in the hospital than necessary, and they tend to have more procedures performed than are medically necessary.

  

  

  

In the city official’s argument, the two boldface portions play which of the following roles?

  1. The first states the conclusion of the city official’s argument; the second provides support for that conclusion.

  2. The first is used to support the conclusion of the city official’s argument; the second states that conclusion.

  3. The first was used to support the conclusion drawn by hospital critics; the second states the position that the city official’s argument opposes.

  4. The first was used to support the conclusion drawn by hospital critics; the second provides support for the conclusion of the city official’s argument.

  5. The first states the position that the city official’s argument opposes; the second states the conclusion of the city official’s argument.

答案:[E]

我选错了。C错的原因在于,第二个BF是卫生官员的结论,而不是官员要反对的。

这两个都是结论,所以BCD排除。E

------------------------------------------------------------------------------------------------------------

GWD-1-Q31:

Which of the following most logically completes the argument below?

 

Davison
     River
farmers are currently deciding between planting winter wheat this fall or spring wheat next spring.  Winter wheat and spring wheat are usually about equally profitable.  Because of new government restrictions on the use of Davison
     River water for irrigation, per acre yields for winter wheat, though not for spring wheat, would be much lower than average.  Therefore, planting spring wheat will be more profitable than planting winter wheat, since‗‗‗‗‗‗.
 

  1. the smaller-than-average size of a winter wheat harvest this year would not be compensated for by higher winter wheat prices

  2. new crops of spring wheat must be planted earlier than the time at which standing crops of winter wheat are ready to be harvested

  3. the spring wheat that farmers in the Davison
          River region plant is well adapted to the soil of the region

  4. spring wheat has uses that are different from those of winter wheat

  5. planting spring wheat is more profitable than planting certain other crops, such as rye

原文意思:冬麦和春麦收益常差不多。新政府的规定使冬麦产量比春麦低。所以种春麦收益好点(指新政府规定后)。原因。。。(实质是找个支持结论的理由)。

答案:因为结论是比较两种麦的收益。其他选项都和两者的收益无关。只有AA说冬麦卖的价格高点,但顶不了产量的减少。春麦的收益还是要好点。

建议:对原文意思的理解,要以原文为依托,不能猜,特别是逻辑题。冬麦价格就是冬麦价格,不是种子。            BCDE划红的都是无关
    [A]

GWD-1-Q33:

A company plans to develop a prototype weeding machine that uses cutting blades with optical sensors and microprocessors that distinguish weeds from crop plants by differences in shade of color.  The inventor of the machine claims that it will reduce labor costs by virtually eliminating the need for manual weeding.

 

 

Which of the following is a consideration in favor of the company’s implementing its plan to develop the prototype?

 

 

  1. There is a considerable degree of variation in shade of color between weeds of different species.

  2. The shade of color of some plants tends to change appreciably over the course of their growing season.

  3. When crops are weeded manually, overall size and leaf shape are taken into account in distinguishing crop plants from weeds.

  4. Selection and genetic manipulation allow plants of virtually any species to be economically bred to have a distinctive shade of color without altering their other characteristics.

  5. Farm laborers who are responsible for the manual weeding of crops carry out other agricultural duties at times in the growing season when extensive weeding is not necessary.

错了     [D]问的问题是:哪种情况有利于该公司实行开发除草机的计划

A显然是一种不利的情况,因为文中说除草机是基于区别野草和庄稼的shade of colors而工作的,而A说:不同种类野草的shade of colors相当不一样,这样自然就对除草机基于shade of colors辨别野草增加了困难。——反之,考虑一种极端情况,如果所有野草都是同一种shade of colors,那么除草机就很容易辨别野草和庄稼了。

D说:通过选种和基因控制,能够让各种庄稼在不改变其他特性的情况下都有一种独特的shade of color,这样自然有利于除草机根据shade of colors的辨别来除草了。

D通过支持证据的方式支持结论。ABC对该计划不利,显然不可能是支持。E错在结论的范围是局限于在割草时因为节省劳动力所以节省成本。E说的是全年的考虑:不割草时可以干别的。所以无关。其实E也有无关标志词。A也有分类错,题目是杂草和庄稼分,而A是杂草之间分。

------------------------------------------------------------------------------------------------------------

GWD-1-Q39:

Which of the following most logically completes the passage below?

 

 

Heavy rains during Centralia’s corn planting season prevented some farmers there from planting corn.  It is now the planting season for soybeans, another of Centralia’s principal crops, and those fields originally intended for corn are dry enough for planting.  Nonetheless, even though soybean prices are unusually high at present, the farmers will leave most of these fields empty rather than plant them with soybeans, since ‗‗‗‗‗‗.

 

 

  1. the extensive rains have led to an increase in the price of corn

  2. some Centralian farmers anticipate serious financial losses due to the extremely wet spring planting season

  3. chemicals that were used to prepare the fields for corn planting would stunt the growth of soybeans

  4. the majority of Centralia’s corn farmers were able to plant corn as they had intended, despite the wet planting season

  5. many Centralian farmers grow both corn and soybeans

[C]

错了。原文说:大雨在C地区的玉米种植季节阻止了农民种植玉米,现在正值种植大豆的季节,大豆是C地区的另外一种主要的农作物,这些原本要种植玉米的地区已经足够干了去种植。然而,尽管大豆的价格是非同寻常的高,农民宁可空着这些地,也不去种植大豆。问原因?

C说这些原来种植玉米地的化学物质将会阻碍大都的生长。(相当于解释原因)B 与原文信息相反 those fields originally intended for corn are dry enough for planting。原文已说这些地足够干了。

------------------------------------------------------------------------------------------------------------

GWD-1-Q40:

Until Now, only injectable vaccines against influenza have been available.  Parents are reluctant to subject children to the pain of injections, but adults, who are at risk of serious complications from influenza, are commonly vaccinated.  A new influenza vaccine, administered painlessly in a nasal spray, is effective for children.  However, since children seldom develop serious complications from influenza, no significant public health benefit would result from widespread vaccination of children using the nasal spray.

 

 

Which of the following is an assumption on which the argument depends?

 

 

  1. Any person who has received the injectable vaccine can safely receive the nasal-spray vaccine as well.

  2. The new vaccine uses the same mechanism to ward off influenza as jnjectable vaccines do.

  3. The injectable vaccine is affordable for all adults.

  4. Adults do not contract influenza primarily from children who have influenza.

  5. The nasal spray vaccine is not effective when administered to adults.
        

E错了。是D短文的结论是小孩用滴鼻药对公众健康没什么太大的意义, 因为小孩得了感冒不会有并发症. 但如果成年人从小孩身上传染到了感冒怎么办呢? 他们还是会有并发症出现, 还是会影响公众健康. 从而否定了愿文的结论.

 E取非是nasal spray对于大人有用,但即使如此也不一定能够很有效改善公众健康,题干结论仍然有成立的可能。


[此贴子已经被作者于2007-5-17 16:12:06编辑过]
沙发
 楼主| 发表于 2007-5-17 15:10:00 | 只看该作者

GWD-2-Question 10:

Environmentalist:  The use of snowmobiles in the vast park north of Milville creates
            
unacceptable levels of air pollution and should be banned.

 

Milville business spokesperson:  Snowmobiling brings many out-of-towners to Milville in winter months, to the great financial benefit of many local residents. So, economics dictate that we put up with the pollution.

 

Environmentalist: I disagree:  A great many cross-country skiers are now kept from visiting Milville by the noise and pollution that snowmobiles generate.

 

Environmentalist responds to the business spokesperson by doing which of the following?

 

A.       Challenging an assumption that certain desirable outcome can derive from only
            
one set of circumstances

B.        Challenging an assumption that certain desirable outcome is outweighed by negative
            
aspects associated with producing that outcome
                

C.       Maintaining that the benefit that the spokesperson desires could be achieved
            
in greater degree by a different means

D.     Claiming that the spokesperson is deliberately misrepresenting the environmentalist’s
            
position in order to be better able to attack it

E.      Denying that an effect that the spokesperson presents as having benefited a certain
            
group of people actually benefited those people

Aonly不对。spokespersonSnowmobiling带来很多人。环境学家说污染让很多人不敢来。环境学家的目的很显然是否定Snowmobiling带来很多人。从而否定给当地居民带来经济利益。即否定这个效果。所以E是合适的,B反了E否认发言人认为会为某特定人群带来好处的那种努力其实不会使那群人受益。E

------------------------------------------------------------------------------------------------------------------------

GWD-2-Question 11:

Many people suffer an allergic reaction to certain sulfites, including those that are commonly added to wine as preservatives.  However, since there are several wine makers who add sulfites to none of the wines they produce, people who would like to drink wine but are allergic to sulfites can drink wines produced by these wine makers without risking an allergic reaction to sulfites.

 

Which of the following is an assumption on which the argument depends?

 

A.        These wine makers have been able to duplicate the preservative effect produced by adding sulfites by means that do not involve adding any potentially allergenic substances to their wine.

B.        Not all forms of sulfite are equally likely to produce the allergic reactions.

C.        Wine is the only beverage to which sulfites are commonly added.

D.       Apart from sulfites, there are no substances commonly present in wine that give rise to an allergic reaction.

E.        Sulfites are not naturally present in the wines produced by these wine makers in amounts large enough to produce an allergic reaction in someone who drinks these wines.

结论是说这些生产者不往酒里加S,所以想喝酒又怕过敏的人可以喝这个,不用冒对S过敏的风险。假设原来酒里自然产生的S的数量不会导致过敏。所以是Ewithout risking an allergic reaction to sulfites
            
说明了酒里肯定有S,只是会引起过敏的程度,如果人工不添加,自然产生的S不会致人过敏。

D因为结论局限于without risking an allergic reaction to sulfites所以D说的其他物质就无关了。
                

抓结论!!!D

------------------------------------------------------------------------------------------------------------------------

GWD-2-Question 12:

Which of the following most logically completes the passage?

 

Concerned about financial well-being of its elderly citizens, the government of Runagia
            
decided two years ago to increase by 20 percent the government-provided pension
            
paid to all Runagians over 65.  Inflation in the intervening period has been negligible,
            
and the increase has been duly received by all eligible Runagians.  Nevertheless,
            
many of them are no better off financially than they were before the increase, in large
            
part because ________.

 

A.        They rely entirely on the government pension for their income

B.        Runagian banks are so inefficient that it can take up to three weeks to cash a
            
pension check

C.        They buy goods whose prices tend to rise especially fast in times of inflation
            

D.       The pension was increased when the number of elderly Runagians below the
            
poverty level reached an all-time high

E.        In Runagia children typically supplement the income of elderly parents, but
            
only by enough to provide them with a comfortable living

Answer:E only by enough揭示了玄机,子女提供的只是刚够用的,如果政府给的多了,子女就会少给,以保持“够用”的水平。题目中已经Inflation 、、 has been negligible所以排除C,题目中又说政策惠及所有老年人,排除D

------------------------------------------------------------------------------------------------------------------------

GWD-2-Question 18:

Last year all refuse collected by Shelbyville city services was incinerated.  This
            
incineration generated a large quantity of residue ash.  In order to reduce the amount
            
of residue ash Shelbyville generates this year to half of last year’s total, the city has
            
revamped its collection program.  This year city services will separate for recycling

enough refuse to reduce the number of truckloads of refuse to be incinerated to half
            
of last year’s number. 

 

Which of the following is required for the revamped collection program to achieve its aim?

 

A.        This year, no materials that city services could separate for recycling will be incinerated.

B.        Separating recyclable materials from materials to be incinerated will cost Shelbyville
            
less than half what it cost last year to dispose of the residue ash.

C.        Refuse collected by city services will contain a larger proportion of recyclable
            
materials this year than it did last year.

D.       The refuse incinerated this year will generate no more residue ash per truckload

incinerated than did the refuse incinerated last year.

E.        The total quantity of refuse collected by Shelbyville city services this year will be

no greater than that collected last year

.原文推理是the number of truckloads of refuse to be incinerated 减少到去年的一半,所以产生的residue ash 也会减少到去年的一半。它的GAP拿去烧的垃圾减少一半,产生的灰也减少一半。C只会影响今年产生的垃圾,并没有填补原文推理的GAP。只有D才填补了这个GAP,即拉去烧的每车垃圾不能产生更多的灰,否则车数少了也没用。E范围太广,是无关,垃圾可以多收,但不一定都去烧并产生灰。
                [D]

------------------------------------------------------------------------------------------------------------

GWD-2-Q12:

Which of the following most logically completes the argument?

 

Yorco and Zortech are two corporations that employ large numbers of full-time workers who are paid by the hour.  Publicly available records indicate that Yorco employs roughly the same number of such hourly wage workers as Zortech does but spends a far higher total sum per year on wages for such workers.  Therefore, hourly wages must be higher, on average, at Yorco than at Zortech, since _____.

 

  1. Zortech spends a higher total sum per year than Yorco does to provide its hourly wage workers with benefits other than wages

  2. the work performed by hourly wage workers at Zortech does not require a significantly higher level of skill than the work performed by hourly wage workers at Yorco does

  3. the proportion of all company employees who are hourly wage workers is significantly greater at Yorco than it is at Zortech

  4. overtime work, which is paid at a substantially higher rate than work done during the regular work week, is rare at both Yorco and Zortech

  5. the highest hourly wages paid at Yorco are higher than the highest hourly wages paid at Zortech

这是个比较,比单位时间收入。两个公司:人数一样,但Y的支付工资多,所以Y的单位工资高,有一个变量就是工作时间,D说高工资的加班极少,就排除了加班时间的差别,排除他因,只能是单位时间工资YZ多。E:与最高工资和B的技能无关。D

------------------------------------------------------------------------------------------------------------

GWD-2-Q14:

GWD-10-Q29:

Smithtown
                    University
’s fund-raisers succeeded in getting donations from 80 percent of the potential donors they contacted.  This success rate, exceptionally high for university fund-raisers, does not indicate that they were doing a good job.  On the contrary, since the people most likely to donate are those who have donated in the past, good fund-raisers constantly try less-likely prospects in an effort to expand the donor base
结论  The high success rate shows insufficient canvassing effort.

 

Which of the following, if true, provides more support for the argument?

 

  1. Smithtown
                            University
    ’s fund-raisers were successful in their contacts with potential donors who had never given before about as frequently as were fund-raisers for other universities in their contacts with such people.

  2. This year the average size of the donations to Smithtown
                            University
    from new donors when the university’s fund-raisers had contacted was larger than the average size of donations from donors who had given to the university before.

  3. This year most of the donations that came to Smithtown
                            University
    from people who had previously donated to it were made without the university’s fund-raisers having made any contact with the donors.

  4. The majority of the donations that fund-raisers succeeded in getting for Smithtown
                            University
    this year were from donors who had never given to the university before.

  5. More than half of the money raised by Smithtown
                            University
    ’s fund-raisers came from donors who had never previously donated to the university.

 Answer: 1.该题争议的根源在于错误理解了原文的结论和不合理的推理(或者称加进自己的信息)。从原文得出的东西必须是MUST BE TRUE,而不是POSSIBLY TRUE

2。该题的结论是:80%的成功率不能说明他们工作做得好(不是:他们工作做不好)。即割断80%成功率对工作做得好的证明力。至于最后一句话是一个补充,因为该句很模糊。

3A提供的信息仅限于S大学的RAISERSNEW DONORS方面和其他大学的RAISERS做得差不多好,并没有突出的地方(本身或和原文结合不能说明80%OLD DONORS比例较大或反之,比如刚开始NEW DONORS总是多,总之,80%NEW DONOR OLD DONORS那个占的比例大是不知道的。这是典型的加进自己信息)

4A对结论的支持表现在:工作做得好,原文提供的标准是EXPAND DONOR BASE(即扩展NEW DONORS)。这方面S大学和他大学的RAISERS差不多(根据A),这80%中,高出别人的部分是OLD DONORS的,而根据原文的标准,这部分不能说明工作做得好。所以这exceptionally high 80%,按照原文的好的标准,其实和其他大学一样,比如他是70%成功率(在说明工作做得好方面,也就是NEW DONORS方面),其它他大学也是70%,所以你不能用80%来证明S大学的RAISERS工作做得好,即支持结论。

5。选项中C为无关项,BDEWEAKENC意思为OLD DONORS中,多数不用CONTACT便捐款。对OLD DONORS是否需CONTACTS才捐款和原文(即评价集资者工作的标准)没关系(不能推出RAISERS没努力
                    A

------------------------------------------------------------------------------------------------------------

GWD-2-Q17:

GWD-10-Q30:

The quality of unrefined olive oil is not actually defined in terms of acidity, yet extensive tests have shown that the less free oleic acid an unrefined olive oil contains per liter, the higher its quality. The proportion of free oleic acid that an olive oil contains is an accurate measure of the oil’s acidity.这个olive oil应该是精练的了。

 

If the statements above are all true, which of the following conclusions is best supported by them?

 

  1. When an olive oil is refined, the concentration of oleic acid in the oil is reduced.

  2. The quality of an unrefined olive oil can be determined only by accurately measuring its acidity.

  3. If an unrefined olive oil is intermediate in acidity between two other unrefined olive oils, it will also be intermediate between them in quality.

  4. Free oleic acid is the only acid that unrefined olive oil contains.

  5. People who judge the quality of unrefined olive oils actually judge those oils by their acidity, which the judges can taste.

觉得那个选项都理由不充足,但倾向A选项,理由如下:

原文三句话:1。实际上,未提炼橄榄油的质量不由酸性来定义

2。但是,大量试验显示,未提炼橄榄油的游离油酸越少,质量越好

3。(提炼后)橄榄油的游离油酸等同于橄榄有的酸性(只有游离油酸,没有其他酸)

推理的结论:4。由12可推出未提炼橄榄油的游离油酸不等同于其酸性,即还有其他酸。

5。由34可推出提炼的过程中,其他酸被去掉。

6。在去掉其他酸的过程中,游离油酸也会相应去掉一些(去酸的过程经常是中和的过程,各种酸都要相应去掉一些)。而且由2可知游离油酸对橄榄油来说,不是好东西,能去掉多少便去掉多少。总上两点,提炼过程中,游离油酸会相应减少。即ACONCENTRATION指浓度)

B错。理由是违反1

C错。理由是由4知未提炼橄榄油中还有其他酸,而酸性取决于游离油酸和其他酸。质量却只和游离油酸有关(或者准确说原文没说出关系)。所以没法比较酸性和质量的相对关系

D错。理由是违反4

E错。原文根本就没这方面的信息

 C

------------------------------------------------------------------------------------------------------------

GWD-2-Q19:

Sunflowers growing in pots were placed, with their roots submerged, in the pond contaminated with radioactive elements.  The sunflowers kept growing; in the process, they absorbed radioactive elements.  Within twelve days, 85 percent of the radioactive elements were removed from the water, which is no less than can be accomplished with the much more expensive conventional filtration techniques.  Scientists therefore propose using sunflowers for decontamination wherever there are radioactively contaminated ponds.

 

Which of the following, if true, points to a limitation on the applicability of the proposed method of decontamination?

 

  1. Some plants other than sunflowers can also remove radioactive elements from water.

  2. The water in many ponds contaminated with radioactive elements is so cold that it would kill sunflowers whose roots were submerged in it.

  3. Sunflowers that grow with their roots submerged in water grow less well than sunflowers growing under optimal conditions on dry land.

  4. Only species of sunflowers with large leaves can have their roots submerged in water and still keep growing.

  5. In ponds in which the circulation of the water is artificially increased, sunflowers absorb radioactive elements far faster than they do in other ponds.

   Answer: B

------------------------------------------------------------------------------------------------------------


[此贴子已经被作者于2007-5-17 15:39:38编辑过]
板凳
 楼主| 发表于 2007-5-17 15:11:00 | 只看该作者

GWD-2-Q21:

Environmental organizations want to preserve the land surrounding the Wilgrinn Wilderness Area from residential development.  They plan to do this by purchasing that land from the farmers who own it.  That plan is ill-conceived:  if the farmers did sell their land, they would sell it to the highest bidder, and developers would outbid any other bidders.  On the other hand, these farmers will never actually sell any of the land, provided that farming it remains viable.  But farming will not remain viable if the farms are left unmodernized, and most of the farmers lack the financial resources modernization requires.  And that is exactly why a more sensible preservation strategy would be to assist the farmers to modernize their farms to the extent needed to maintain viability.

 

In the argument as a whole, the two boldface proportions play which of the following roles?

 

  1. The first presents a goal that the argument rejects as ill-conceived; the second is evidence that is presented as grounds for that rejection.

  2. The first presents a goal that the argument concludes cannot be attained; the second is a reason offered in support of that conclusion.

  3. The first presents a goal that the argument concludes can be attained; the second is a judgment disputing that conclusion.

  4. The first presents a goal, strategies for achieving which are being evaluated in the argument; the second is a judgment providing a basis for the argument’s advocacy of a particular strategy.

  5. The first presents a goal that the argument endorses; the second presents a situation that the argument contends must be changed if that goal is to be met in the foreseeable future.

   Answer:

D:前面提了个总目标,可以达到

B是因为没有仔细的看完题目和选项

BThe first presents a goal that the argument concludes cannot be attained

这个明显和原文最后:And that is exactly why a more sensible preservation strategy would be...矛盾,原文不认为那个目标是不可达到的

A 错在
        
第一句是全文的目标
        
且是作者想要达成的
        
作者reject的是 by purchasing land 这个strategy

E 错在后面一句the second presents a situation that the argument contends must be changed if that goal is to be met in the foreseeable future.

must be changed的是by purchasing land 这个strategy

these farmers will never actually sell any of the land, provided that farming it remains viable 这句粗体
                
只是作者对于by purchasing land 这个strategy 所作出的 judgment 之一
            
而且作者还希望farming it remains viable 持续
                
才不会被residential development占领

我想这题阅读是最大问题
            
我也读了好久
            
而且很多细节要注意D

------------------------------------------------------------------------------------------------------------

GWD-2-Q40:

Which of the following most logically completes the argument?

 

A new drug, taken twice daily for one month, is an effective treatment for a certain disease.  The drug now most commonly prescribed for the disease occasionally has serious side effects such as seizures; in field tests, the new drug’s side effects, though no worse than mild nausea, turned out to be much more frequent.  Nevertheless, the new drug is clearly preferable as a treatment, since _____.

 

A.      people who experience nausea are prone to discontinue use of the new drug prematurely

B.     

C.     

D.    

E.      there is a nonprescription medication that when taken with the new drug prevents the onset of nausea

答案是e,因为new drug可以和非处方药一起服用而预防nausea [E]

------------------------------------------------------------------------------------------------------------

GWD-3-Q2:

GWD-4-Q21:

Hunter:  Hunters alone are blamed for the decline in Greenrock
                        National Forest
’s deer population over the past ten years.  Yet clearly, black bears have also played an important role in this decline.  In the past ten years, the forest’s protected black bear population has risen sharply, and examination of black bears found dead in the forest during the deer hunting season showed that a number of them had recently fed on deer.

 

In the hunter’s argument, the boldface portion plays which of the following roles?

 

  1. It is the main conclusion of the argument.

  2. It is an objection that has been raised against the main conclusion of the argument.

  3. It is a judgment that the argument opposes.

  4. It is a finding that the argument seeks to explain.

  5. It provides evidence in support of the main conclusion of the argument.

[A]

鹿群的减少finding. 黑熊在其中扮演了重要角色则是hunter's conclusion.

------------------------------------------------------------------------------------------------------------

GWD-3-Q8:

Which of the following most logically completes the passage?

 

Garnet and RenCo each provide health care for their employees.  Garnet pays for both testing of its employees’ cholesterol levels and treatment of high cholesterol.  This policy saves Garnet money, since high cholesterol left untreated for many years leads to conditions that require very expensive treatment.  However, RenCo dose not have the same financial incentive to adopt such a policy, because ______.

 

A.      early treatment of high cholesterol dose not entirely eliminate the possibility of a stroke later in life

B.      the mass media regularly feature stories encouraging people to maintain diets that are low in cholesterol

C.      RenCo has significantly more employees than Garnet has

D.     RenCo’s employees are unlikely to have higher cholesterol levels than Garnet’s employees

E.      the average length of time an employee stays with RenCo is less than it is with Garnet
                

[E]

在单位呆的时间长短对公司是否采用该法有影响,所以E对,虽然没讲界限,但对问题没影响,因为说出原因不需要充分性。请注意DE的区别。一个胆固醇高点低点有何影响原文没信息,一个时间长点短点有影响。

题目说明两家公司的策略不同
            
AB属于identical choice 没法说明两家公司的差别
            
排除

------------------------------------------------------------------------------------------------------------

GWD-3-Q9:

GWD-5-Q31:

GWD-8-Q14:

Studies in restaurants show that the tips left by customers who pay their bill in cash tend to be larger when the bill is presented on a tray that bears a credit-card logo.  Consumer psychologists hypothesize that simply seeing a credit-card logo makes many credit-card holders willing to spend more because it reminds them that their spending power exceeds the cash they have immediately available.

 

Which of the following, if true, most strongly supports the psychologists’ interpretation of the studies?

 

  1. The effect noted in the studies is not limited to patrons who have credit cards.

  2. Patrons who are under financial pressure from their credit-card obligations tend to tip less when presented with a restaurant bill on a tray with credit-card logo than when the tray has no logo.

  3. In virtually all of the cases in the studies, the patrons who paid bills in cash did not possess credit cards.

  4. In general, restaurant patrons who pay their bills in cash leave larger tips than do those who pay by credit card.

  5. The percentage of restaurant bills paid with given brand of credit card increases when that credit card’s logo is displayed on the tray with which the bill is prepared.

B是通过加强原文证据对结论的证明力来加强结论。即B说明看到LOGO会让人想到信用卡的状况,从而影响付TIPS行为。如果信用卡的状况是不好,则付TIPS少(B所说),如果信用卡的状况好,则付TIPS多(原文证据),是一个规律的两个方面。所以加强了结论。

几类特殊的支持题可以预测答案:

前提和结论无密切关系的:直接支持结论

因果:文章给出两件事,说一件导致另一件的,支持的答案为:无因无果或有因有果;排除它因;因果没有颠倒;因的来源没有问题

充分必要:就是结论带条件的(充分必要指示词见lawyer的文章),答案为充分条件存在,必要条件一定存在。

类比:两类事物具可比性

调查:调查数据充分,具代表性

OG的解释非常强调文章中两者的关系,relationship。本题一样,其实文章就是想建立起两者之间的relationshipLOGO会影响人们的消费行为。LOGO和消费之间有relationship。既然存在固定的ralationship,就会出现同比上升和同比下降两种情况,所以加强经常是给出同比上升或同比下降中文章所没有提到的那一种。

------------------------------------------------------------------------------------------------------------

GWD-3-Q16:

GWD-4-Q16:

Economist:  Tropicorp, which constantly seeks profitable investment opportunities, has been buying and clearing sections of tropical forest for cattle ranching, although pastures newly created there become useless for grazing after just a few years.  The company has not gone into rubber tapping, even though greater profits can be made from rubber tapping, which leaves the forest intact.  Thus, some environmentalists conclude that
                    Tropicorp has not acted wholly out of economic self-interest.  However, these environmentalists are probably wrong.  The initial investment required for a successful rubber-tapping operation is larger than that needed for a cattle ranch.  Furthermore, there is a shortage of workers employable in rubber-tapping operations, and finally, taxes are higher on profits from rubber tapping than on profits from cattle ranching.

 

In the economist’s argument, the two boldfaced portions play which of the following roles?

 

  1. The first supports the conclusion of the economist’s argument; the second calls that conclusion into question.

  2. The first states the conclusion of the economist’s argument; the second supports that conclusion.

  3. The first supports the environmentalists’ conclusion; the second states that conclusion.

  4. The first states the environmentalists’ conclusion; the second states the conclusion of the economist’s argument.

  5. Each supports the conclusion of the economist’s argument.

[C]

  

------------------------------------------------------------------------------------------------------------

GWD-3-Q17:

GWD-4-Q15:

Brochure:  Help conserve our city’s water supply.  By converting the landscaping in your yard to a water-conserving landscape, you can greatly reduce your outdoor water use.  A water-conserving landscape is natural and attractive, and it also saves you money.

 

Criticism:  For most people with yards, the savings from converting to a water-conserving landscape cannot justify the expense of new landscaping, since typically the conversion would save less than twenty dollars on a homeowner’s yearly water bills.

 

Which of the following, if true, provides the best basis for a rebuttal of the criticism?

 

  1. Even homeowners whose yards do not have water-conserving landscapes can conserve water by installing water-saving devices in their homes.

  2. A conventional landscape generally requires a much greater expenditure on fertilizer and herbicide than does a water-conserving landscape.

  3. A significant proportion of the residents of the city live in buildings that do not have yards.

  4. It costs no more to put in water-conserving landscaping than it does to put in conventional landscaping.

  5. Some homeowners use more water to maintain their yards than they use for all other purposes combined.


                
关键词没看清楚

------------------------------------------------------------------------------------------------------------

GWD-3-Q18:

GWD-4-Q26:

Which of following most logically completes the argument?

 

The last members of a now-extinct species of a European wild deer called the giant dear lived in Ireland about 16,000 years ago.  Prehistoric cave paintings in France depict this animal as having a large hump on its back.  Fossils of this animal, however, do not show any hump.  Nevertheless, there is no reason to conclude that the cave paintings are therefore inaccurate in this regard, since ______.

 

  1. some prehistoric cave paintings in France also depict other animals as having a hump

  2. fossils of the giant deer are much more common in Ireland than in France

  3. animal humps are composed of fatty tissue, which dose not fossilize

  4. the cave paintings of the giant deer were painted well before 16,000 years ago

  5. only one currently existing species of deer has any anatomical feature that even remotely resembles a hump

[C]

 

------------------------------------------------------------------------------------------------------------

GWD-3-Q20:

Shipping Clerk:  The five specially ordered shipments sent out last week were sent out on Thursday.  Last week, all of the shipments that were sent out on Friday consisted entirely of building supplies, and the shipping department then closed for the weekend.  Four shipments were sent to Truax Construction last week, only three of which consisted of building supplies.

 

If the shipping clerk’s statements are true, which of the following must also be true?

 

  1. At least one of the shipments sent to Truax Construction last week was specially ordered.

  2. At least one of last week’s specially ordered shipments did not consist of building supplies.

  3. At least one of the shipments sent to Truax Construction was not sent out on Thursday of last week.

  4. At least one of the shipments that were sent out on Friday of last week was sent to Truax Construction.

  5. At least one of the shipments sent to Truax Construction last week was sent out before Friday.

[E]

A不一定,因为并没有说明剩下的一个是什么

B 不知道其它时间如周一或周四等送出的有没有BS

C 也不一定,也可能周二的货也有BS

D 不一定,也可能其它时间发的BS

E 周五送出的全都是BS,所以有一个不是BS的,肯定是周五之前发的。

因为从原文只知周五SHIPMENT的都包括building supplies不知其他时间SHIPMENT是否包括building supplies,所以去TC的四个SHIPMENT不知何时送出(除知一个不是周五,因为没有building supplies),也有可能除周五外的其他时间,因为也可能其它时间SHIPMENT的有building supplies。因为不知何时送出,所以也不知至少有一个周四送出,何况周四送出也不一定是specially orderedspecially ordered都周四送出,不等于周四送出的都是specially ordered)。所以A和原文信息还差远,不可能是MUST BE。又因为不知其他时间的SHIPMENT是否有building supplies,所以B也错。至于AB是否相当,我认为两者差别太大了,谈不上相当。

------------------------------------------------------------------------------------------------------------

GWD-3-Q31:

GWD-4-Q32:

Although most smoking-related illnesses are caused by inhaling the tar in tobacco smoke, it is addiction to nicotine that prevents most smokers from quitting.  In an effort to decrease the incidence of smoking-related illnesses, lawmakers in Sandonia plan to reduce the average quantity of nicotine per cigarette by half over the next five years.  Unfortunately, smokers who are already addicted to nicotine tend to react to such reductions by smoking correspondingly more cigarettes.

 

The information above most strongly supports which of the following predictions about the effects of implementing the Sandonian government’s plan?

 

  1. The average quantity of tar inhaled by Sandonian smokers who are currently addicted to nicotine will probably not decrease during the next five years.

  2. Sandonian smokers who are not already addicted to nicotine will probably also begin to smoke more cigarettes during the next five years than they had previously.

  3. The annual number of Sandonian smokers developing smoking-related illnesses will probably decrease during the next five years.

  4. The proportion of Sandonians attempting to quit smoking who succeed in that attempt will probably decrease during the next five years.

  5. The number of Sandonians who quit smoking during the next five years will probably exceed the number who quit during the last five years.

[A]

本文只提已上瘾的人,未上瘾的无关

------------------------------------------------------------------------------------------------------------

GWD-3-Q32:

GWD-4-Q27:

Newspaper editorial:

 

In an attempt to reduce the crime rate, the governor is getting tough on criminals and making prison conditions harsher.  Part of this effort has been to deny inmates the access they formerly had to college-level courses.  However, this action is clearly counter to the governor’s ultimate goal, since after being released form prison, inmates who had taken such courses committed far fewer crimes overall than other inmates.

 

Which of the following is an assumption on which the argument depends?

 

  1. Not being able to take college-level courses while in prison is unlikely to deter anyone from a crime that he or she might otherwise have committed.

  2. Former inmates are no more likely to commit crimes than are members of the general population.

  3. The group of inmates who chose to take college-level courses were not already less likely than other inmates to commit crimes after being released.

  4. Taking high school level courses in prison has less effect on an inmate’s subsequent behavior than taking college-level courses does.

  5. The governor’s ultimate goal actually is to gain popularity by convincing people that something effective is being done about crime.

C The group of inmates who chose to take college-level courses were not already less likely than other inmates to commit crimes after being released. 就是说这帮修大学课程的家伙进来的时候和那帮不要修大学课程的人一样危险,一样的犯罪可能性和倾向, 这样的话,原文的结论修大学课程有用就成立了,因为这帮原来都一样混的家伙修了大学课程后,斯文了很多,比没修课程的人文明了不少。

CNOT按掉,The group of inmates who chose to take college-level courses were (not) already less likely than other inmates to commit crimes after being released.

这帮修课的人进监狱的时候就已经很斯文了,那不就是说修课没有用么,削弱了原文.  

------------------------------------------------------------------------------------------------------------

GWD-3-Q38:

Kate:  The recent decline in numbers of the Tennessee warbler, a North American songbird that migrates each fall to coffee plantations in South America, is due to the elimination of the dense tree cover that formerly was a feature of most South American coffee plantations.

 

Scott:  The population of the spruce budworm, the warbler’s favorite prey in North America, has been dropping.  This is a more likely explanation of the warbler’s decline.

 

Which of the following, if true, most seriously calls Scott’s hypothesis into question?

 

  1. The numbers of the Baltimore oriole, a songbird that does not eat budworms but is as dependent on South American coffee plantations as is the Tennessee warbler, are declining.

  2. The spruce-budworm population has dropped because of a disease that can infect budworms but not Tennessee warblers.

  3. The drop in the population of the spruce budworm is expected to be only temporary.

  4. Many Tennessee warbler have begun migrating in the fall to places other than traditional coffee plantations.

  5. Although many North American songbirds have declined in numbers, no other species has experienced as great a decline as has the Tennessee warbler.

[A]异因同果

  

------------------------------------------------------------------------------------------------------------


[此贴子已经被作者于2007-5-17 15:42:22编辑过]
地板
 楼主| 发表于 2007-5-17 15:11:00 | 只看该作者

GWD-3-Q40:

GWD-4-Q38:

Two centuries ago, Tufe
    Peninsula became separated form the mainland, isolating on the newly formed Tufe
     Island a population of Turfil sunflowers.  This population’s descendants grow to be, on average, 40 centimeters shorter than Turfil sunflowers found on the mainland.  Tufe
    Island is significantly drier than Tufe
     Peninsula was.  So the current average height of Tufe’s Turfil sunflowers is undoubtedly at least partially attributable to changes in Tufe’s environmental conditions.

 

 

Which of the following is an assumption on which the argument depends?

 

 

  1. There are no types of vegetation on Tufe
          Island that are known to benefit from dry conditions.

  2. There were about as many Turfil sunflowers on Tufe
         Peninsula two centuries ago as there are on Tufe
          Island today.

  3. The mainland’s environment has not changed in ways that have resulted in Turfil sunflowers on the mainland growing to be 40 centimeters taller than they did two centuries ago.

  4. The soil on Tufe
          Island, unlike that on the mainland, lacks important nutrients that help Turfil sunflowers survive and grow tall in a dry environment.

  5. The 40-centimeter height difference between the Turfil sunflowers on Tufe
          Island and those on the mainland is the only difference between the two populations.

[C]

200年前岛屿分化分离之前,两个地方的条件是一样的,从而排除了他因导致sunflower变得更矮小,进一步说明确实是分离后岛屿的环境改变才导致这种高度不一的

即:大陆的情况没变化(高度都差不多,至少差不到40以上),是海岛的环境变了,所以才导致高度差别大了。

------------------------------------------------------------------------------------------------------------

GWD-4-Q8:

From 1980 to 1989, total consumption of fish in the country of Jurania increased by 4.5 percent, and total consumption of poultry products there increased by 9.0 percent.  During the same period, the population of Jurania increased by 6 percent, in part due to immigration to Jurania from other countries in the region.

 

 

If the statements above are true, which of the following must also be true on the basis of them?

 

 

  1. During the 1980’s in Jurania, profits of wholesale distributors of poultry products increased at a greater rate than did profits of wholesale distributors of fish.

  2. For people who immigrated to Jurania during the 1980’s, fish was less likely to be a major part of their diet than was poultry.

  3. In 1989 Juranians consumed twice as much poultry as fish.

  4. For a significant proportion of Jurania’s population, both fish and poultry products were a regular part of their diet during the 1980’s.

  5. Per capita consumption of fish in Jurania was lower in 1989 than in 1980.

1.魚的消費量增加4.5%   

2.家禽的消費量增加9%

3.人口增加6%           13条就可推出E

 [E]

B的错误在于For people who immigrated to Jurania during the 1980’s, fish was less likely to be a major part of their diet than was poultry.说的是量,而题干中说的是比率。比率高,不一定量就大。

假设原来人有n个现在是n*(1+6%)

原来消费鱼m条现在是m*(1+4.5%)

原来人均消费鱼m/n

现在人均消费鱼1.045m/1.06n                        这是一类题

显然人均消费鱼少了                 

------------------------------------------------------------------------------------------------------------

GWD-4-Q14:           错了。

GWD-5-Q2:

Guidebook writer:  I have visited hotels throughout the country and have noticed that in those built before 1930 the quality of the original carpentry work is generally superior to that in hotels built afterward.  Clearly carpenters working on hotels before 1930 typically worked with more skill, care, and effort than carpenters who have worked on hotels built subsequently.

 

 

Which of the following, if true, most seriously weakens the guidebook writer’s argument?

 

 

  1. The quality of original carpentry in hotels is generally far superior to the quality of original carpentry in other structures, such as houses and stores.

  2. Hotels built since 1930 can generally accommodate more guests than those built before 1930.

  3. The materials available to carpenters working before 1930 were not significantly different in quality from the materials available to carpenters working after 1930.

  4. The better the quality of original carpentry in a building, the less likely that building is to fall into disuse and be demolished.

  5. The average length of apprenticeship for carpenters has declined significantly since 1930.

D就是说,1930年以前的木工也是有好有坏,但是能留到今天的都是当初的好的木工,不好的都disusedemolish了,所以不能以现在留下来的这些1930年以前的直接与1930年以后的做比较,削弱了原文的逻辑。E is not relevant, because there is not necessarily any direct relationship between the "length of apprenticeship" and the "quality of the carpentry work".

我选A比较对象与原文不符。

[D]

------------------------------------------------------------------------------------------------------------

GWD-4-Q20:

GWD-5-Q40:

GWD-11-Q8:

Community activist:  If Morganville wants to keep its central shopping district healthy, it should prevent the opening of a huge SaveAll discount department store on the outskirts of Morganville.  Records from other small towns show that whenever SaveAll has opened a store outside the central shopping district of a small town, within five years the town has experienced the bankruptcies of more than a quarter of the stores in the shopping district.

 

 

The answer to which of the following would be most useful for evaluating the community activist’s reasoning?

 

 

  1. Have community activists in other towns successfully campaigned against the opening of a SaveAll store on the outskirts of their towns?

  2. Do a large percentage of the residents of Morganville currently do almost all of their shopping at stores in Morganville?

  3. In towns with healthy central shopping districts, what proportion of the stores in those districts suffer bankruptcy during a typical five-year period?

  4. What proportion of the employees at the SaveAll store on the outskirts of Morganville will be drawn from Morganville?

  5. Do newly opened SaveAll stores ever lose money during their first five years of operation?

[C]

关键理解什麽是EVALUATION题。EVALUATION题是WEAKENSUPPORT的结合,对选项的YESNO或两个极端的回答能对原文的结论起WEAKENSUPPORT的作用。注意:不是YES就一定要起WEAKENSUPPORT作用,而是一个起WEAKEN作用,另一个就起SUPPORT作用,反之亦然。即不同的回答能起不同的作用。

对本题,两个极端的回答是0%100%。如果正常破产率为0%,则支持原文结论(开折扣店有害),如果是100%破产率,则削弱原文(开折扣店没有害,1/4破产率不算高)。故起EVALUATION的作用。

 

 

本题简单做法:题干的more than a quarter 与选项的what proportion形成对应。

GWD-5-Q3:

GWD-11-Q2:

The average hourly wage of television assemblers in Vernland has long been significantly lower than that in neighboring Borodia.  Since Borodia dropped all tariffs on Vernlandian televisions three years ago, the number of televisions sold annually in Borodia has not changed.  However, recent statistics show a drop in the number of television assemblers in Borodia.  Therefore, updated trade statistics will probably indicate that the number of televisions Borodia imports annually from Vernland has increased.

 

Which of the following is an assumption on which the argument depends?

 

  1. The number of television assemblers in Vernland has increased by at least as much as the number of television assemblers in Borodia has decreased.

  2. Televisions assembled in Vernland have features that televisions assembled in Borodia do not have.

  3. The average number of hours it takes a Borodian television assembler to assemble a television has not decreased significantly during the past three years.

  4. The number of televisions assembled annually in Vernland has increased significantly during the past three years.

  5. The difference between the hourly wage of television assemblers in Vernland and the hourly wage of television assemblers in Borodia is likely to decrease in the next few years.

[C]

提干的结论是:因为工人数量减少,推出进口电视机数量增加。即B国生产电视机数量减少。

人数*工作效率*工作时间=总产量。

这个公式可以推出,工作时间少了,如果要想使总产量少,就要保持工作效率不变,否则工作效率提高了,产量还是会增加。

------------------------------------------------------------------------------------------------------------

GWD-5-Q8:

When a new restaurant, Martin’s Cafe, opened in Riverville last year, many people predicted that business at the Wildflower Inn, Riverville’s only other restaurant, would suffer from the competition.  Surprisingly, however, in the year since Martin’s Cafe opened, the average number of meals per night served at the Wildflower Inn has increased significantly.

 

Which of the following, if true, most helps to explain the increase?

 

  1. Unlike the Wildflower Inn, Martin’s Cafe serves considerably more meals on weekends than it does on weekdays.

  2. Most of the customers of Martin’s Cafe had never dined in Riverville before this restaurant opened, and on most days Martin’s Cafe attracts more customers than it can seat.

  3. The profit per meal is higher, on average, for meals served at Martin’s Cafe than for those served at the Wildflower Inn.

  4. The Wildflower Inn is not open on Sundays, and therefore Riverville residents who choose to dine out on that day must either eat at Martin’s Cafe or go to neighboring towns to eat.

  5. A significant proportion of the staff at Martin’s Cafe are people who formerly worked at the Wildflower Inn and were hired away by the owner of Martin’s Cafe.

BM店就餐的人在M店开张之前大多都没在R地吃过饭(说明M的开张增加了总的吃饭人数),而且M店总是有人抢不到位置吃饭(说明增加人数有部分没吃到饭)——》只好到W店去吃了

------------------------------------------------------------------------------------------------------------

GWD-5-Q13:

It is true of both men and women that those who marry as young adults live longer than those who never marry.  This does not show that marriage causes people to live longer, since, as compared with other people of the same age, young adults who are about to get married have fewer of the unhealthy habits that can cause a person to have a shorter life, most notably smoking and immoderate drinking of alcohol.

 

Which of the following, if true, most strengthens the argument above?错两次

 

  1. Marriage tends to cause people to engage less regularly in sports that involve risk of bodily harm.

  2. A married person who has an unhealthy habit is more likely to give up that habit than a person with the same habit who is unmarried.

  3. A person who smokes is much more likely than a nonsmoker to marry a person who smokes at the time of marriage, and the same is true for people who drink alcohol immoderately.

  4. Among people who marry as young adults, most of those who give up an unhealthy habit after marriage do not resume the habit later in life.

  5. Among people who as young adults neither drink alcohol immoderately nor smoke, those who never marry live as long as those who marry.

本题没看明白:原文第一句话是一个事实,第二句开始是conclusion,即:婚姻不是导致长寿的原因strengthen的是这个conclusionE充分地strengthen这个结论。而B/D都是削弱,证明婚姻导致长寿。

拉拉的C无关

[E]

------------------------------------------------------------------------------------------------------------

GWD-5-Q15:

GWD-11-Q9:

Lightbox, Inc., owns almost all of the movie theaters in Washington
                        County
and has announced plans to double the number of movie screens it has in the county within five years.  Yet attendance at Lightbox’s theaters is only just large enough for profitability now and the county’s population is not expected to increase over the next ten years.  Clearly, therefore, if there is indeed no increase in population, Lightbox’s new screens are unlikely to prove profitable.

 

Which of the following, if true about Washington
                        County
, most seriously weakens the argument?

 

  1. Though little change in the size of the population is expected, a pronounced shift toward a younger, more affluent, and more entertainment-oriented population is expected to occur.

  2. The sales of snacks and drinks in its movie theaters account for more of Lightbox’s profits than ticket sales do.

  3. In selecting the mix of movies shown at its theaters, Lightbox’s policy is to avoid those that appeal to only a small segment of the moviegoing population.

  4. Spending on video purchases, as well as spending on video rentals, is currently no longer increasing.

  5. There are no population centers in the county that are not already served by at least one of the movie theaters that Lightbox owns and operates.

没看清,如果说会有赢利,他因说有其它观众群即可。A題目的焦點是: 看電影"人口"的問題!

虽然人口没有增加,但电影消费群扩大了,断桥削弱
                    [A]


[此贴子已经被作者于2007-5-17 15:49:26编辑过]
5#
 楼主| 发表于 2007-5-17 15:11:00 | 只看该作者

GWD-5-Q16:
            

GWD-13-Q14:

Maize contains the vitamin niacin, but not in a form the body can absorb.  Pellagra is a disease that results from niacin deficiency.  When maize was introduced into southern Europe from the Americas in the eighteenth century, it quickly became a dietary staple, and many Europeans who came to subsist primarily on maize developed pellagra.  Pellagra was virtually unknown at that time in the Americas, however, even among people who subsisted primarily on maize.

 

Which of the following, if true, most helps to explain the contrasting incidence of pellagra described above?

 

  1. Once introduced into southern Europe, maize became popular with landowners because of its high yields relative to other cereal crops.

  2. Maize grown in the Americas contained more niacin than maize grown in Europe did.

  3. Traditional ways of preparing maize in the Americas convert maize’s niacin into a nutritionally useful form.

  4. In southern Europe many of the people who consumed maize also ate niacin-rich foods.

  5. Before the discovery of pellagra’s link with niacin, it was widely believed that the disease was an infection that could be transmitted from person to person.

[C]

没看明白,题目要解释为什么EuropeanAmerican都吃maize,为什么E得病,而A不得病。

前提提到得P病的原因是n的不足,而n不足是因为m里面的n不可以吸收。因此可以推出如果n可以吸收的话,那么就不会有P了。

C就解释了用传统方法做mn就可以吸收,所以A就没有病了。同时暗含E没有用传统方法做m

------------------------------------------------------------------------------------------------------------

GWD-5-Q19:
            
B

GWD-7-Q28:

GWD-11-Q13:

Historian:  In the Drindian Empire, censuses were conducted annually to determine the population of each village.  Village census records for the last half of the 1600’s are remarkably complete.  This very completeness makes one point stand out; in five different years, villages overwhelmingly reported significant population declines.  Tellingly, each of those five years immediately followed an increase in a certain Drindian tax.  This tax, which was assessed on villages, was computed by the central government using the annual census figures.  Obviously, whenever the tax went up, villages had an especially powerful economic incentive to minimize the number of people they recorded; and concealing the size of a village’s population from government census takers would have been easy.  Therefore, it is reasonable to think that the reported declines did not happen.

 

In the historian’s argument, the two portions in boldface play which of the following roles?

 

  1. The first supplies a context for the historian’s argument; the second acknowledges a consideration that has been used to argue against the position the historian seeks to establish.

  2. The first presents evidence to support the position that the historian seeks to establish; the second acknowledges a consideration that has been used to argue against that position.

  3. The first provides a context for certain evidence that supports the position that the historian seeks to establish; the second is that position.

  4. The first is a position for which the historian argues; the second is an assumption that serves as the basis of that argument.

  5. The first is an assumption that the historian explicitly makes in arguing for a certain position; the second acknowledges a consideration that calls that assumption into question.

整段话都是historian的观点。
            
第二个BFhistorian的中心论点。(排除A2,文章前半段全是中心论点的推理过程。第一个BF是推理的起点。 3,如果缺乏第一个BF,即records不完整,则推理不成立。因为如果records不完整,则可能1600s中大部分年都有population decline,而其中只有五年followed an increase in a certain Drindian tax,这样的话就不能推出人口下降是由于避税而虚报的。

这一段话整个是historian的一个argument,前后看来的对立其实是表面的。粗看题目容易理解成历史学家说人口下降而作者通过分析提出人口并未下降,导致倾向选A。事实不是如此,A说的第二部分意在推翻historian的结论并不存在,因为整个reasoning都是历史学家的,第二部分正是他的结论本身。[C]

补充一点,再次读A选项The first supplies a context for the historian’s argument;C选项The first provides a context for certain evidence that supports the position that the historian seeks to establish;会发现其中的细微差别,是这个选项中很关键的。

其实第一个画线内容就是一个事实,并且它是第二画线句的推理过程的一个环节,期支持作用

我觉得这道题目的意义在于,对于BF题目,除了转承关系词,找到作者支持的结论,以及这个结论用的前提,作者反对的结论,以及这个反对结论用的前提外,还要注意哪一些是背景的东西,或者说是属于大家公用的前提的东西。               这道题目就是考了一个公用的前提。

这个题有个快捷方法.第二个黑体部分明显是结论.所以从选项的尾部看:

ABE都说是consideration.明显不对..

D:说是assumption.也不对..

只有C:说是position. 结论..

------------------------------------------------------------------------------------------------------------

GWD-5-Q20:

Scientists typically do their most creative work before the age of forty.  It is commonly thought that this happens because aging by itself brings about a loss of creative capacity.  However, studies show that a disproportionately large number of the scientists who produce highly creative work beyond the age of forty entered their field at an older age than is usual.  Since by the age of forty the large majority of scientists have been working in their field for at least fifteen years, the studies’ finding strongly suggests that the real reason why scientists over forty rarely produce highly creative work is not that they have simply aged but rather that they generally have spent too long in a given field.

 

In the argument given, the two portions in boldface play which of the following roles?

 

  1. The first is the position that the argument as a whole opposes; the second is an objection that has been raised against a position defended in the argument.

  2. The first is a claim that has been advanced in support of a position that the argument opposes; the second is a finding that has been used in support of that position.

  3. The first is an explanation that the argument challenges; the second is a finding that has been used in support of that explanation.

  4. The first is an explanation that the argument challenges; the second is a finding on which that challenge is based.

  5. The first is an explanation that the argument defends; the second is a finding that has been used to challenge that explanation.

[D]

这道题目就是因果结构。

第一句话就说明了结果:40岁之前做出的成就多。然后给了两个原因:一个是年龄问题(作者反对的解释),另外一个是加入这个行业时间长短问题(作者支持的解释)。

从这个结构上就很清楚地看出来应该选择D

所以我觉得在做BF题的时候也要注意一下文章的结构。

------------------------------------------------------------------------------------------------------------

GWD-5-Q21:

In Teruvia, the quantity of rice produced per year is currently just large enough to satisfy domestic demand.  Teruvia’s total rice acreage will not be expanded in the foreseeable future, nor will rice yields per acre increase appreciably.  Teruvia’s population, however, will be increasing significantly for years to come.  Clearly, therefore, Teruvia will soon have to begin importing rice.

 

Which of the following is an assumption on which the argument depends?

 

  1. No pronounced trend of decreasing per capita demand for rice is imminent in Teruvia.

  2. Not all of the acreage in Teruvia currently planted with rice is well suited to the cultivation of rice.

  3. None of the strains of rice grown in Teruvia are exceptionally high-yielding.

  4. There are no populated regions in Teruvia in which the population will not increase.

  5. There are no major crops other than rice for which domestic production and domestic demand are currently in balance in Teruvia.

A明显是答案,E是无关,不要说题外的作物

[A]

------------------------------------------------------------------------------------------------------------

GWD-5-Q26:

GWD-11-Q21:

Healthy lungs produce a natural antibiotic that protects them from infection by routinely killing harmful bacteria on airway surfaces.  People with cystic fibrosis, however, are unable to fight off such bacteria, even though their lungs produce normal amounts of the antibiotic.  The fluid on airway surfaces in the lungs of people with cystic fibrosis has an abnormally high salt concentration; accordingly, scientists hypothesize that the high salt concentration is what makes the antibiotic ineffective.

 

Which of the following, if true, most strongly supports the scientists’ hypothesis?

 

  1. When the salt concentration of the fluid on the airway surfaces of healthy people is raised artificially, the salt concentration soon returns to normal.

  2. A sample of the antibiotic was capable of killing bacteria in an environment with an unusually low concentration of salt.

  3. When lung tissue from people with cystic fibrosis is maintained in a solution with a normal salt concentration, the tissue can resist bacteria.

  4. Many lung infections can be treated by applying synthetic antibiotics to the airway surfaces.

  5. High salt concentrations have an antibiotic effect in many circumstances.

[C]

BC易混。B中的unusually low concentration不好,不如C

------------------------------------------------------------------------------------------------------------

GWD-5-Q28:

Wind farms, which generate electricity using arrays of thousands of wind-powered turbines, require vast expanses of open land.  County
                X and County
                Y have similar terrain, but the population density of County
                X is significantly higher than that of County
                    Y.
  Therefore, a wind farm proposed for one of the two counties should be built in County
                Y rather than in County
                    X.

 

Which of the following, if true, most seriously weakens the planner’s argument?

 

  1. County
                        X and County
                            Y
    are adjacent to each other, and both are located in the windiest area of the state.

  2. The total population of County
                        Y is substantially greater than that of County
                            X.

  3. Some of the electricity generated by wind farms in County
                            Y
    would be purchased by users outside the county.

  4. Wind farms require more land per unit of electricity generated than does any other type of electrical-generation facility.

  5. Nearly all of County
                        X’s population is concentrated in a small part of the county, while County
                            Y
    ’s population is spread evenly throughout the country.

[E]

x的人口(虽然多于y)都集中在很小的地方,所以空地可能很多。

------------------------------------------------------------------------------------------------------------

GWD-5-Q29:

Over the past five years, the price gap between name-brand cereals and less expensive store-brand cereals has become so wide that consumers have been switching increasingly to store brands despite the name brands’ reputation for better quality.  To attract these consumers back, several manufacturers of name-brand cereals plan to narrow the price gap between their cereals and store brands to less than what it was five years ago.

 

Which of the following, if true, most seriously calls into question the likelihood that the manufacturers’ plan will succeed in attracting back a large percentage of consumers who have switched to store brands?

 

  1. There is no significant difference among manufacturers of name-brand cereals in the prices they charge for their products.

  2. Consumers who have switched to store-brand cereals have generally been satisfied with the quality of those cereals.

  3. Many consumers would never think of switching to store-brand cereals because they believe the name brand cereals to be of better quality.

  4. Because of lower advertising costs, stores are able to offer their own brands of cereals at significantly lower prices than those charged for name-brand cereals.

  5. Total annual sales of cereals—including both name-brand and store-brand cereals—have not increased significantly over the past five years.

原文的推理是:name-brand cereals 生产商计划用将差价缩小的方法将丢失的顾客吸引回来(价格仍然比STORE高,说明主要仍要靠质量吸引顾客,降价只是消除STORE的优势,消除顾客买STORE谷物的原因----理解这点很重要)(方法目的型)

DSTORE有能力提供低价对name-brand cereals 要缩小差价的方法起不了什麽作用,因为STORE价格本来就低,你说STORE有能力提供低价不等于白说吗。没有削弱作用

Bname-brand cereals 生产商想要象原来一样靠质量吸引他们,现在这个因素不存在了(顾客现在对STORE的质量满意,不考虑NAME优势)。吸引他们的因素没了。顾客有何理由回去。价格吗?(质量已经不重要)name-brand cereals 生产商价格降价后还是比STORE高。

------------------------------------------------------------------------------------------------------------

GWD-5-Q30:

GWD-11-Q12:     模糊性大C  E 存疑

Which of the following most logically completes the argument?

 

The irradiation of food kills bacteria and thus retards spoilage.  However, it also lowers the nutritional value of many foods.  For example, irradiation destroys a significant percentage of whatever vitamin B1 a food may contain.  Proponents of irradiation point out that irradiation is no worse in this respect than cooking.  However, this fact is either beside the point, since much irradiated food is eaten raw, or else misleading, since _______.

 

  1. many of the proponents of irradiation are food distributors who gain from food’s having a longer shelf life

  2. it is clear that killing bacteria that may be present on food is not the only effect that irradiation has

  3. cooking is usually the final step in preparing food for consumption, whereas irradiation serves to ensure a longer shelf life for perishable foods

  4. certain kinds of cooking are, in fact, even more destructive of vitamin B1 than carefully controlled irradiation is

  5. for food that is both irradiated and cooked, the reduction of vitamin B1 associated with either process individually is compounded

E有道理,但不能作为MISLEADING的论据. E 倾向于表明无法分清是DAMAGE 营养更多. 如果原文改为, or else insufficient,since--------------.E为较好的答案.

C 明显misleading. proponent之所以把cooking拉出来比,就是想找个替罪羊, C 选项明确说出,两者并不同时存在,一个是保鲜,一个是烧, 保鲜过程对营养的破坏是不能扯到后期的cook 上的.

所以我觉得C 在逻辑上完成了ARGUMENT

作者抨击现有的保存食物的方法——irradiation,并给出了理由指明其缺点,相对应的肯定存在有更好的保持食物的方法,而那些支持irradiation的人们,比如生产食物的农民觉得irradiation成本相对低,指出irradiation的缺点只是很小的缺点,即与cooking相比irradiation损失的B1一样少

这明显是那些支持irradiation的人们在误导读者,现有的保存食物的方法——irradiation只能与更好的保存方法相比,怎么能与cooking相比呢?!irradiation只损失一点点的B1,而新的保持方法可以不损失B1,那当然还是要舍弃irradiation而用新的,与cooking损失多少B1有什么关系呢?这就是文章最后讲this fact is misleading的原因。

所以只要指出irradiationcooking根本就不是一回事不具有可比性,就可以了。

因为Irradiation Cooking 的破坏VB1的作用是幂数增加的(即(1-x%)^n,类似于利息),而FACT则仅是Cooking onlyIrradiation only的比较。Irradiation支持者举出FACT来支持他们的论点时,忽略选项E这个事实,所以misleading

不同意C,因为it's off the topic of “this repect”: destroy VB1

proponents的意思用通俗一点的方法讲出来大家就明白了:你们别看irradiation对营养有破坏作用,其实它对营养的破坏还没cooking大呢!——你们平时做菜不都要cook的么?所以不管你们用不用irradiation,这些营养最终都会被cooking破坏。因此irradiation并不会给你们带来额外的营养损失!

http://forum.chasedream.com/dispbbs.asp?boardid=24&star=9&replyid=497679&id=56351&skin=0&page=1 [E]

 

------------------------------------------------------------------------------------------------------------


[此贴子已经被作者于2007-5-17 15:51:54编辑过]
6#
 楼主| 发表于 2007-5-17 15:11:00 | 只看该作者

GWD-5-Q32:

Which of the following most logically completes the argument?

 

 

Although the pesticide TDX has been widely used by fruit growers since the early 1960’s, a regulation in force since 1960 has prohibited sale of fruit on which any TDX residue can be detected.  That regulation is about to be replaced by one that allows sale of fruit on which trace amounts of TDX residue are detected.  In fact, however, the change will not allow more TDX on fruit than was allowed in the 1960’s, because ______.

 

 

  1. pre-1970 techniques for detecting TDX residue could detect it only when it was present on fruit in more than the trace amounts allowed by the new regulations

  2. many more people today than in the 1960’s habitually purchase and eat fruit without making an effort to clean residues off the fruit

  3. people today do not individually consume any more pieces of fruit, on average, than did the people in the 1960’s

  4. at least a small fraction of the fruit sold each year since the early 1960’s has had on it greater levels of TDX than the regulation allows

  5. the presence of TDX on fruit in greater than trace amounts has not been shown to cause any harm even to children who eat large amounts of fruit

[A]

以前,监测到有RESIDUE就不能卖,新规定允许有trace amounts 。(虽然说条件放宽了),但较松的规定却没有让更多的苹果通过监测。A说的是原来的技术其实单纯的RESIDUE是检查不出来的,只有到了TDX的量高于trace amounts ,才能检查成有RESIDUE。这样虽然条件放宽了,但技术水平使实际检查到的TDX的量是差不多的。所以说较松的规定却没有让更多的苹果通过监测。

------------------------------------------------------------------------------------------------------------

GWD-5-Q33:

Wolves generally avoid human settlements.  For this reason, domestic sheep, though essentially easy prey for wolves, are not usually attacked by them.  In Hylantia prior to 1910, farmers nevertheless lost considerable numbers of sheep to wolves each year.  Attributing this to the large number for wolves, in 1910 the government began offering rewards to hunters for killing wolves.  From 1910 to 1915, large numbers of wolves were killed.  Yet wolf attacks on sheep increased significantly.

 

 

Which of the following, if true, most helps to explain the increase in wolf attacks on sheep?

 

 

  1. Populations of deer and other wild animals that wolves typically prey on increased significantly in numbers from 1910 to 1915.

  2. Prior to 1910, there were no legal restrictions in Hylantia on the hunting of wolves.

  3. After 1910 hunters shot and wounded a substantial number of wolves, thereby greatly diminishing these wolves’ ability to prey on wild animals.

  4. Domestic sheep are significantly less able than most wild animals to defend themselves against wolf attacks.

  5. The systematic hunting of wolves encouraged by the program drove many wolves in Hylantia to migrate to remote mountain areas uninhabited by humans.

[C]

CD好,

------------------------------------------------------------------------------------------------------------

GWD-5-Q38:

GWD-11-Q20:   错了

Nitrogen dioxide is a pollutant emitted by automobiles.  Catalytic converters, devices designed to reduce nitrogen dioxide emissions, have been required in all new cars in Donia since 1993, and as a result, nitrogen dioxide emissions have been significantly reduced throughout most of the country.  Yet although the proportion of new cars in Donia’s capital city has always been comparatively high, nitrogen dioxide emissions there have showed only an insignificant decline since 1993.

 

 

Which of the following, if true, most helps to explain the insignificant decline in nitrogen dioxide emissions in Donia’s capital city?

 

 

  1. More of the cars in Donia’s capital city were made before 1993 than after 1993.

  2. The number of new cars sold per year in Donia has declined slightly since 1993.

  3. Pollutants other than nitrogen dioxide that are emitted by automobiles have also been significantly reduced in Donia since 1993.

  4. Many Donians who own cars made before 1993 have had catalytic converters installed in their cars.

  5. Most car trips in Donia’s capital city are too short for the catalytic converter to reach its effective working temperature.

[E]

比如1993年前11万辆,199310万辆。那末要求1993年后的车装CONVERTER,肯定污染会明显减少。A解释不了原文的现象。E短程,达不到CC工作需要的温度,也就不能减少污染。

GWD-5-Q38:

GWD-11-Q20:   错了

Nitrogen dioxide is a pollutant emitted by automobiles.  Catalytic converters, devices designed to reduce nitrogen dioxide emissions, have been required in all new cars in Donia since 1993, and as a result, nitrogen dioxide emissions have been significantly reduced throughout most of the country.  Yet although the proportion of new cars in Donia’s capital city has always been comparatively high, nitrogen dioxide emissions there have showed only an insignificant decline since 1993.

 

 

Which of the following, if true, most helps to explain the insignificant decline in nitrogen dioxide emissions in Donia’s capital city?

 

 

  1. More of the cars in Donia’s capital city were made before 1993 than after 1993.

  2. The number of new cars sold per year in Donia has declined slightly since 1993.

  3. Pollutants other than nitrogen dioxide that are emitted by automobiles have also been significantly reduced in Donia since 1993.

  4. Many Donians who own cars made before 1993 have had catalytic converters installed in their cars.

  5. Most car trips in Donia’s capital city are too short for the catalytic converter to reach its effective working temperature.

[E]

比如1993年前11万辆,199310万辆。那末要求1993年后的车装CONVERTER,肯定污染会明显减少。A解释不了原文的现象。E短程,达不到CC工作需要的温度,也就不能减少污染。

GWD-5-Q38:

GWD-11-Q20:   错了

Nitrogen dioxide is a pollutant emitted by automobiles.  Catalytic converters, devices designed to reduce nitrogen dioxide emissions, have been required in all new cars in Donia since 1993, and as a result, nitrogen dioxide emissions have been significantly reduced throughout most of the country.  Yet although the proportion of new cars in Donia’s capital city has always been comparatively high, nitrogen dioxide emissions there have showed only an insignificant decline since 1993.

 

 

Which of the following, if true, most helps to explain the insignificant decline in nitrogen dioxide emissions in Donia’s capital city?

 

 

  1. More of the cars in Donia’s capital city were made before 1993 than after 1993.

  2. The number of new cars sold per year in Donia has declined slightly since 1993.

  3. Pollutants other than nitrogen dioxide that are emitted by automobiles have also been significantly reduced in Donia since 1993.

  4. Many Donians who own cars made before 1993 have had catalytic converters installed in their cars.

  5. Most car trips in Donia’s capital city are too short for the catalytic converter to reach its effective working temperature.

[E]

比如1993年前11万辆,199310万辆。那末要求1993年后的车装CONVERTER,肯定污染会明显减少。A解释不了原文的现象。E短程,达不到CC工作需要的温度,也就不能减少污染。

GWD-6-Q4:

In the past the country of Siduria has relied heavily on imported oil.  Siduria recently implemented a program to convert heating systems from oil to natural gas.  Siduria already produces more natural gas each year than it burns, and oil production in Sidurian oil fields is increasing at a steady pace.  If these trends in fuel production and usage continue, therefore, Sidurian reliance on foreign sources for fuel should decline soon.

 

Which of the following is an assumption on which the argument depends?

 

  1. In Siduria the rate of fuel consumption is rising no more quickly than the rate of fuel production.

  2. Domestic production of natural gas is rising faster than is domestic production of oil in Siduria.

  3. No fuel other than natural gas is expected to be used as a replacement for oil in Siduria.

  4. Buildings cannot be heated by solar energy rather than by oil or natural gas.

  5. All new homes that are being built will have natural-gas-burning heating systems.

   Answer: A

------------------------------------------------------------------------------------------------------------

GWD-6-Q14:

Political Advertisement:

 

Mayor Delmont’s critics complain about the jobs that were lost in the city under Delmont’s leadership.  Yet the fact is that not only were more jobs created than were eliminated, but the average pay for these new jobs has been higher than the average pay for jobs citywide every year since Delmont took office.  So there can be no question that throughout Delmont’s tenure the average paycheck in this city has been getting steadily bigger.

 

Which of the following, if true, most strengthens the argument in the advertisement?

 

  1. The average pay for jobs created in the city during the past three years was higher than the average pay for jobs created in the city earlier in Mayor Delmont’s tenure.

  2. Average pay in the city was at a ten-year low when Mayor Delmont took office.

  3. Some of the jobs created in the city during Mayor Delmont’s tenure have in the meantime been eliminated again.

  4. The average pay for jobs eliminated in the city during Mayor Delmont’s tenure has been roughly equal every year to the average pay for jobs citywide.

  5. The average pay for jobs in the city is currently higher than it is for jobs in the suburbs surrounding the city.

错了仅仅对比了新增工作机会和就任以来全市平均收入,如果不对比新增工作机会和减少工作机会的平均收入哪个高(A),仍然有可能总收入是下降的。新增的工作收入高,所以只要去掉的工作收入一样,总平均收入还是高的。所以DAD任期的后几年和D任期的前几年的新增工资水平的比较,不能得出工资总水平的结论。

原文用两个证据more jobs created new jobs has been higher than the average 去证明结论average paycheck has been getting steadily bigger。如果取消的工作也比平均工资高,那末原文的第二个证据对结论的证明力就减少。D排除这种情况。所以加强了第二个证据对结论的证明力。故为选项D

------------------------------------------------------------------------------------------------------------

GWD-6-Q15:

Capuchin monkeys often rub their bodies with a certain type of millipede.  Laboratory tests show that secretions from the bodies of these millipedes are rich in two chemicals that are potent mosquito repellents, and mosquitoes carry parasites that debilitate capuchins.  Some scientists hypothesize that the monkeys rub their bodies with the millipedes because doing so helps protect them from mosquitoes.

 

Which of the following, if true, provides the most support for the scientists’ hypothesis?

 

  1. A single millipede often gets passed around among several capuchins, all of whom rub their bodies with it.

  2. The two chemicals that repel mosquitoes also repel several other varieties of insects.

  3. The capuchins rarely rub their bodies with the millipedes except during the rainy season, when mosquito populations are at their peak.

  4. Although the capuchins eat several species of insects, they do not eat the type of millipede they use to rub their bodies.

  5. The two insect-repelling chemicals in the secretions of the millipedes are carcinogenic for humans but do not appear to be carcinogenic for capuchins.

   Answer: C
                    

------------------------------------------------------------------------------------------------------------

GWD-6-Q16:

Historian:  Newton developed mathematical concepts and techniques that are fundamental to modern calculus.  Leibniz developed closely analogous concepts and techniques.  It has traditionally been thought that these discoveries were independent.  Researchers have, however, recently discovered notes of Leibniz’ that discuss one of Newton’s books on mathematics.  Several scholars have argued that since the book includes a presentation of Newton’s calculus concepts and techniques, and since the notes were written before Leibniz’ own development of calculus concepts and techniques, it is virtually certain that the traditional view is false.  A more cautious conclusion than this is called for, however.  Leibniz’ notes are limited to early sections of Newton’s book, sections that precede the ones in which Newton’s calculus concepts and techniques are presented.

 

In the historian’s reasoning, the two boldfaced portions play which of the following roles?

 

  1. The first provides evidence in support of the overall position that the historian defends; the second is evidence that has been used to support an opposing position.

  2. The first provides evidence in support of the overall position that the historian defends; the second is that position.

  3. The first provides evidence in support of an intermediate conclusion that is drawn to provide support for the overall position that the historian defends; the second provides evidence against that intermediate conclusion.

  4. The first is evidence that has been used to support a conclusion that the historian criticizes; the second is evidence offered in support of the historian’s own position.

  5. The first is evidence that has been used to support a conclusion that the historian criticizes; the second is further information that substantiates that evidence.

错了。原文有三个观点, a. 传统的观点: 牛顿和莱布尼兹的研究是各自独立得出的。 b. 一些学者的观点: 莱布尼兹是受牛顿影响。因为莱布尼兹在得出自己的研究前,参考过牛顿的数学书。 c. 作者的观点: 作者反对观点b, 也就是说支持观点a, 因为莱布尼兹只参考了牛顿书的前面的章节,而牛顿的理论是在后面的章节才出现。

再看答案,应该很清楚是D.  错误在于没搞懂这几个观点的关系。A more cautious conclusion肯定是把上面的观点反对掉了,而不是继承。我误以为只有两个观点。

------------------------------------------------------------------------------------------------------------

GWD-6-Q20:

Five years ago, as part of a plan to encourage citizens of Levaska to increase the amount of money they put into savings, Levaska’s government introduced special savings accounts in which up to $3,000 a year can be saved with no tax due on the interest unless money is withdrawn before the account holder reaches the age of sixty-five.  Millions of dollars have accumulated in the special accounts, so the government’s plan is obviously working.

 

Which of the following, if true, most seriously weakens the argument?

 

  1. A substantial number of Levaskans have withdrawn at least some of the money they had invested in the special accounts.

  2. Workers in Levaska who already save money in long-term tax-free accounts that are offered through their workplace cannot take advantage of the special savings accounts introduced by the government.

  3. The rate at which interest earned on money deposited in regular savings accounts is taxed depends on the income bracket of the account holder.

  4. Many Levaskans who already had long-term savings have steadily been transferring those savings into the special accounts.

  5. Many of the economists who now claim that the government’s plan has been successful criticized it when it was introduced.

错了。要注意结论:即政策是为了吸引更多的存款,而D说存款只是换了个帐户,总量并没有增加。我选A,已经取出的钱是无关的。The conclusion is whether the approach has encouraged the saving. If the conclusion is that everyone will benefit, B will be the answer. D

------------------------------------------------------------------------------------------------------------


[此贴子已经被作者于2007-5-17 15:55:51编辑过]
7#
 楼主| 发表于 2007-5-17 15:11:00 | 只看该作者

GWD-6-Q21:

An overwhelming proportion of the most productive employees at SaleCo’s regional offices work not eight hours a day, five days a week, as do other SaleCo employees, but rather ten hours a day, four days a week, with Friday off.  Noting this phenomenon, SaleCo’s president plans to increase overall productivity by keeping the offices closed on Fridays and having all employees work the same schedule—ten hours a day, four days a week.

 

Which of the following, if true, provides the most reason to doubt that the president’s plan, if implemented, will achieve its stated purpose?

 

  1. Typically, a SaleCo employee’s least productive hours in the workplace are the early afternoon hours.

  2. None of the employees who work four days a week had volunteered to work that schedule, but all were assigned to it by their supervisors.

  3. Working ten hours a day has allowed the most productive employees to work two hours alone each day in their respective offices relatively undisturbed by fellow employees.

  4. Employees at SaleCo are compensated not on the basis of how many hours a week they work but on the basis of how productive they are during the hours they are at work.

  5. Those SaleCo employees who have a four-day workweek do not take any of their office work to do at home on Fridays.

错了。因为这些最高效员工的高效得益于额外工作的那两个小时里,办公室比较安静,不受打扰,而如果人人都10*4工作制,那么“undisturbed”这样一个benefit就不存在了。

前提:某公司有两种上班方式:一部分人是5天,每天8小时。另一部分人是4天,每天10个小时

结论:为提高总的生产率,老板决定全部改为4天,每天10小时(改过来才使效率变化,选项必须反映这种改变。这也是结论的特殊性)。所以,正确选项必须反映这两种制度的区别并说明这种改变会导致总生产率下降,也就是生产率提高的条件不存在了。

A。下午工作效率最低。两种制度都有下午,反映不了差别。无关

B。这反映了一种上班方式的特点,但不知另一种的情况,不知换过来之后总效率是高了,还是低了。不明。不过好像有关,先放着。

C。每天工作10小时让效率最高的职员每天有两个小时不受同事干扰。就是说其他8小时是受干扰而影响效率的。这样两种上班方式一合并就10个小时都受干扰了。当然影响效率。正是答案。放弃B再往下看是否有更合适的。

D。不能说明两种制度差别。无关

E。和效率无关。无关。

至此,选出C为答案。

1、看出作者的结论是:老板们希望通过改变作息来提高效率。(要猜测老板的心思)

2、要看出选项C中的条件是原来作息时间中所特有的现象,改变了作息时间,也就改变了这个条件,使得结论成立的基础动摇了。(要敏感于两种作息制度产生的结果)

想法:从“most productive”员工与别人不一样的工作方法出发考虑。

------------------------------------------------------------------------------------------------------------

GWD-6-Q28:

Which of the following most logically completes the argument below?

 

According to promotional material published by the city of Springfield, more tourists stay in hotels in Springfield than stay in the neighboring city of Harristown.  A brochure from the largest hotel in Harristown claims that more tourists stay in that hotel than stay in the Royal Arms Hotel in Springfield.  If both of these sources are accurate, however, the county’s “Report on Tourism” must be in error in indicating that _______.

 

  1. more tourists stay in hotel accommodations in Harristown than stay in the Royal Arms Hotel

  2. the Royal Arms Hotel is the only hotel in Springfield

  3. there are several hotels in Harristown that are larger than the Royal Arms Hotel

  4. some of the tourists who have stayed in hotels in Harristown have also stayed in the Royal Arms Hotel

  5. some hotels in Harristown have fewer tourist guests each year than the Royal Arms Hotel has

第一,   S市住旅馆的总游客能比H市多,第二,H市某旅馆客人比S市某旅馆的客人多。得出结论:S市就需要有其他旅馆,所以B所说的RA只是S的唯一一家旅馆是不可能的。

此类题目最好先自己推出结论那个什么report根本就是个无聊的东东B

------------------------------------------------------------------------------------------------------------

GWD-6-Q32:

Proposal:  Carbon dioxide and methane in the atmosphere block the escape of heat into space.  So emission of these “greenhouse” gases contributes to global warming.  In order to reduce global warming, emission of greenhouse gases needs to be reduced.  Therefore, the methane now emitted from open landfills should instead be burned to produce electricity.

 

Objection:  The burning of methane generates carbon dioxide that is released into the atmosphere.

 

Which of the following, if true, most adequately counters the objection made to the proposal?

 

  1. Every time a human being or other mammal exhales, there is some carbon dioxide released into the air.

  2. The conversion of methane to electricity would occur at a considerable distance from the landfills.

  3. The methane that is used to generate electricity would generally be used as a substitute for a fuel that does not produce any greenhouse gases when burned.

  4. Methane in the atmosphere is more effective in blocking the escape of heat from the Earth than is carbon dioxide.

  5. The amount of methane emitted from the landfills could be reduced if the materials whose decomposition produces methane were not discarded, but recycled.

 答案中必须体现二者的关系。E说如果将分解甲烷的原料循环使用,产生的甲烷会减少。这个明显不在原文的逻辑推理范围当中,和反对者的话没关系C说的是M将替代某种不会产生二氧化碳的燃料。燃料的替代与否跟本文章一点关系都没有,是无关选项。本文是在讨论如何减少温室效应不是讨论燃料。

D说甲烷比二氧化碳产生的温室效应更厉害,也就是说燃烧还是能减少温室效应的。D

------------------------------------------------------------------------------------------------------------

GWD-6-Q33:

Crowding on Mooreville’s subway frequently leads to delays, because it is difficult for passengers to exit from the trains.  Subway ridership is projected to increase by 20 percent over the next 10 years.  The Metroville Transit Authority plans to increase the number of daily train trips by only 5 percent over the same period.  Officials predict that this increase is sufficient to ensure that the incidence of delays due to crowding does not increase.

 

Which of the following, if true, provides the strongest grounds for the officials’ prediction?

 

  1. By changing maintenance schedules, the Transit Authority can achieve the 5 percent increase in train trips without purchasing any new subway cars.

  2. The Transit Authority also plans a 5 percent increase in the number of bus trips on routes that connect to subways.

  3. For most commuters who use the subway system, there is no practical alternative public transportation available.

  4. Most of the projected increase in ridership is expected to occur in off-peak hours when trains are now sparsely used.

  5. The 5 percent increase in the number of train trips can be achieved without an equal increase in Transit Authority operational costs.

   Answer: D

------------------------------------------------------------------------------------------------------------

GWD-6-Q38:

Three large companies and seven small companies currently manufacture a product with potential military applications.  If the government regulates the industry, it will institute a single set of manufacturing specifications to which all ten companies will have to adhere.  In this case, therefore, since none of the seven small companies can afford to convert their production lines to a new set of manufacturing specifications, only the three large companies will be able to remain in business.

 

Which of the following is an assumption on which the author’s argument relies?

 

  1. None of the three large companies will go out of business if the government does not regulate the manufacture of the product.

  2. It would cost more to convert the production lines of the small companies to a new set of manufacturing specifications than it would to convert the production lines of the large companies.

  3. Industry lobbyists will be unable to dissuade the government from regulating the industry.

  4. Assembly of the product produced according to government manufacturing specifications would be more complex than current assembly procedures.

  5. None of the seven small companies currently manufactures the product to a set of specifications that would match those the government would institute if the industry were to be regulated.

B取非,则变成:小厂的转换成本不比大厂的转换成本高,但这并不能削弱结论,因为根据题目中的已知条件,即使转换成本相同,也有可能大厂能够承担而小厂承担不了。所以,B不是Assumption.

E取非,则变成:有些小厂目前生产的产品就能够符合政府将要实施的规定,这样就削弱了以后只能三个大厂才能留在这个行业的结论。所以,EAssumption.

 -----------------------------------------------------------------------------------------------------------

GWD-6-Q40:

It is theoretically possible that bacteria developed on Mars early in its history and that some were carried to Earth by a meteorite.  However, strains of bacteria from different planets would probably have substantial differences in protein structure that would persist over time, and no two bacterial strains on Earth are different enough to have arisen on different planets.  So, even if bacteria did arrive on Earth from Mars, they must have died out.

 

The argument is most vulnerable to which of the following criticisms?

 

  1. It fails to establish whether bacteria actually developed on Mars.

  2. It fails to establish how likely it is that Martian bacteria were transported to Earth.

  3. It fails to consider whether there were means other than meteorites by which Martian bacteria could have been carried to Earth.

  4. It fails to consider whether all bacteria now on Earth could have arisen from transported Martian bacteria.

  5. It fails to consider whether there could have been strains of bacteria that originated on Earth and later died out.

 Answer: D是正确的!如果地球上所有的细菌全部来自火星,当然不可能有两种不同的结构,IF这个成立,那么文章的结论说来自火星的灭绝了就错了!因为现有的细菌来自火星,但却没有灭!
                


[此贴子已经被作者于2007-5-17 15:57:23编辑过]
8#
 楼主| 发表于 2007-5-17 15:11:00 | 只看该作者

GWD-7-Q5:

Exposure to certain chemicals commonly used in elementary schools as cleaners or pesticides causes allergic reactions in some children.  Elementary school nurses in Renston report that the proportion of schoolchildren sent to them for treatment of allergic reactions to those chemicals has increased significantly over the past ten years.  Therefore, either Renston’s schoolchildren have been exposed to greater quantities of the chemicals, or they are more sensitive to them than schoolchildren were ten years ago.

 

Which of the following is an assumption on which the argument depends?

 

  1. The number of school nurses employed by Renston’s elementary schools has not decreased over the past ten years.

  2. Children who are allergic to the chemicals are no more likely than other children to have allergies to other substances.

  3. Children who have allergic reactions to the chemicals are not more likely to be sent to a school nurse now than they were ten years ago.

  4. The chemicals are not commonly used as cleaners or pesticides in houses and apartment buildings in Renston.

  5. Children attending elementary school do not make up a larger proportion of Renston’s population now than they did ten years ago.

E比较的是学校内部的比例,原文讲的是Elementary school nurses in Renston repots that..., 说的就是在Renston地区的小学,而E的意思是说在Renston地区上小学的人数并不比10年前所占人口的比例大,问题在于E说的是一个比例,而不是一个绝对数,有可能现在总的地区人口数下降,而上小学的人数不变,可所占总人口数的比例还增高了(如E,

我觉得如果E说的是现在上小学的人数不比10年前上小学的人数多,则也是一个取非削弱。

E的错误是把比例当成了绝对数C

------------------------------------------------------------------------------------------------------------

GWD-7-Q14:

GWD-8-Q19:

Certain politicians in the country of Birangi argue that a 50 percent tax on new automobiles would halt the rapid increase of automobiles on Birangi’s roads and thereby slow the deterioration of Birangi’s air quality.  Although most experts agree that such a tax would result in fewer Birangians buying new vehicles and gradually reduce the number of automobiles on Birangi’s roads, they contend that it would have little impact on Birangi’s air-quality problem.

 

Which of the following, if true in Birangi, would most strongly support the experts’ contention about the effect of the proposed automobile tax on Birangi’s air-quality problem?

 

  1. Automobile emissions are the largest single source of air pollution.

  2. Some of the proceeds from the new tax would go toward expanding the nonpolluting commuter rail system.

  3. Currently, the sales tax on new automobiles is considerably lower than 50 percent.

  4. Automobiles become less fuel efficient and therefore contribute more to air pollution as they age.

  5. The scrapping of automobiles causes insignificant amounts of air pollution.

正确答案D:虽然新车进入少,但现有的车随着老化,污染更严重,所以污染仍然大,仍然没有好转。故SUPPORT专家观点。

A。无关。车辆是否是最大污染源与专家观点无关

B。无关,最好是WEAKEN。扩大地铁系统也许会使车辆减少减少量大些,从而使空气污染少些。

C。无关

E。无关,说的是拆旧车。D

------------------------------------------------------------------------------------------------------------

GWD-7-Q17:

Of patients over 65 years old who survived coronary bypass surgery—a procedure widely prescribed for people with heart disease—only 75 percent benefited from the surgery.  Thus it appears that for one in four such patients, the doctors who advised them to undergo this surgery, with its attendant risks and expense, were more interested in an opportunity to practice their skills and in their fee than in helping the patient.

 

Which of the following, if true, most seriously undermines the argument?

 

  1. Many of the patients who receive coronary bypass surgery are less than 55 years old.

  2. Possible benefits of coronary bypass surgery include both relief from troubling symptoms and prolongation of life.

  3. Most of the patients in the survey decided to undergo coronary bypass surgery because they were advised that the surgery would reduce their risk of future heart attacks.

  4. The patients over 65 years old who did not benefit from the coronary bypass surgery were as fully informed as those who did benefit from the surgery as to the risks of the surgery prior to undergoing it.

  5. The patients who underwent coronary bypass surgery but who did not benefit from it were medically indistinguishable, prior to their surgery, from the patients who did benefit.

D的意思为受益和没受益的的两批人都被告知手术的危险性。而结论是医生对没受益的动机不纯。显然DWEAKEN不了结论。还有点加强医生动机不纯,医生完全可以说一套做一套。

E的意思为:受益和没收益两批人手术前在医疗上是分不出的。既然分不出,医生就无法对一批人动机好,对另一批人动机不好。

------------------------------------------------------------------------------------------------------------

GWD-7-Q18:  

GWD-8-Q21:

Although the earliest surviving Greek inscriptions written in an alphabet date from the eighth century B.C., a strong case can be made that the Greeks actually adopted alphabetic writing at least two centuries earlier.  Significantly, the text of these earliest surviving Greek inscriptions sometimes runs from right to left and sometimes from left to right.  Now, the Greeks learned alphabetic writing from the Phoenicians, and in the process they would surely have adopted whatever convention the Phoenicians were then using with respect to the direction of writing.  Originally, Phoenician writing ran in either direction, but by the eighth century B.C. it had been consistently written from right to left for about two centuries.

 

In the argument given, the two portions in boldface play which of the following roles?

 

  1. The first is the position that the argument seeks to establish; the second reports a discovery that has been used to support a position that the argument opposes.

  2. The first is the position that the argument seeks to establish; the second presents an assumption on which the argument relies.

  3. The first presents evidence that is used in support of the position that the argument seeks to establish; the second presents an assumption on which the argument relies.

  4. The first is an objection raised against a position that the argument opposes; the second is the position that the argument seeks to establish.

  5. The first is an objection raised against a position that the argument opposes; the second is evidence that has been used to support that position.

拉拉理解:本题前后没有转折,所以从AB中选。A是发现,是事实,B是假设,文章说:希腊人最早写作题字是八世纪前,但可能表明是在更早,这些表明更早的是从两个方向写的,现在的希腊文是从P学来的,最初P人也是两个方向的,持续了两个世纪,到八世纪前,改成一个方向了,所以,第二个BF就成了一个假设,即希腊人继承了P人的任何写字习惯,否则无法认定更早两个世纪的题字是希腊人的。搭桥

would也是假设的一种提示?

------------------------------------------------------------------------------------------------------------

GWD-7-Q21:  比较搞

Which of the following most logically completes the passage?

 

Each species of moth has an optimal body temperature for effective flight, and when air temperatures fall much below that temperature, the moths typically have to remain inactive on vegetation for extended periods, leaving them highly vulnerable to predators.  In general, larger moths can fly faster than smaller ones and hence have a better chance of evading flying predators, but they also have higher optimal body temperatures, which explains why ______.

 

  1. large moths are generally able to maneuver better in flight than smaller moths

  2. large moths are proportionally much more common in warm climates than in cool climates

  3. small moths are more likely than large moths to be effectively camouflaged while on vegetation

  4. large moths typically have wings that are larger in proportion to their body size than smaller moths do

  5. most predators of moths prey not only on several different species of moth but also on various species of other insects

每个品种的飞蛾对于effective flight飞行有个最佳温度,当空气温度远低于此值时, 飞蛾趴在蔬菜上长期不动,这样使得它们容易被predator攻击.总的来说, 大飞蛾比小飞蛾飞的快,因而更容易逃生, 但是这些大飞蛾需要较高的最佳温度, 这可以解释为什么大飞蛾在温暖气候比寒冷的气候下相应地较多.(b)

如果直选,都不像答案,B相对正确。

but they also have higher optimal body temperatures, which explains why ______. 填空题关键找和空格前内容相符的内容。就是最紧密联系的,别的不用考虑太多

------------------------------------------------------------------------------------------------------------

GWD-7-Q27: 

A significant number of complex repair jobs carried out by Ace Repairs have to be reworked under the company’s warranty.  The reworked jobs are invariably satisfactory.  When initial repairs are inadequate, therefore, it is not because the mechanics lack competence; rather, there is clearly a level of focused concentration that complex repairs require that is elicited more reliably by rework jobs than by first-time jobs.

跳跃修饰不错,但是我认为加and就错了,concentration在两个定从中地位不同,前者是做宾语,后者做主语,所以两个定从不是平行结构,可以将concentration that complexs repairs require中的定从和concentration之间的关系看成修饰词和核心词的关系,作为一个短语,而后面的定从是修饰前面短语中的核心词。语法问题

The argument above assumes which of the following?

 

  1. There is no systematic difference in membership between the group of mechanics who do first-time jobs and the group of those who do rework jobs.

  2. There is no company that successfully competes with Ace Repairs for complex repair jobs.

  3. Ace Repairs’ warranty is good on first-time jobs but does not cover rework jobs.

  4. Ace Repairs does not in any way penalize mechanics who have worked on complex repair jobs that later had to be reworked.

  5. There is no category of repair jobs in which Ace Repairs invariably carries out first-time jobs satisfactorily.

AR作的大多数维修工作必须重修,重修很令人满意。首次修理不太充分,不是修理人员没水平,而是要求的侧重点不同,二修要求高点。结论两个TAHT并列修饰LEVEL

There is no systematic difference in membership between the group of mechanics who do first-time jobs and the group of those who do rework jobs.(A取非,即两组修理工间有系统差别,则削弱题中所述一修不充分,不是因为修理工缺乏能力。因此选A)

------------------------------------------------------------------------------------------------------------

GWD-7-Q30:

Printwell’s Ink Jet Division manufactures ink-jet printers and the ink cartridges they use.  Sales of its ink-jet printers have increased.  Monthly revenues from those sales, however, have not increased, because competition has forced Printwell to cut the prices of its printers.  Unfortunately, Printwell has been unable to bring down the cost of manufacturing a printer.  Thus, despite the increase in printer sales, the Ink Jet Division must be providing the company with much smaller than it used to.

 

Which of the following, if true, most seriously weakens the argument?

 

  1. Ink-jet printers in regular use frequently need new ink cartridges, and Printwell’s printers only accept Printwell’s ink cartridges.

  2. Unlike some competing companies, Printwell sells all of its printers through retailers, and these retailers’ costs account for a sizable proportion of the printers’ ultimate retail price.

  3. Some printer manufacturers have been forced to reduce the sale price of their ink-jet printers even more than Printwell has.

  4. In the past year, no competing manufacturer of ink-jet printers has had as great an increase in unit sales of printers as Printwell has.

  5. In the past year, sales of Printwell’s ink-jet printers have increased more than sales of any other type of printer made by Printwell.

简单!削弱“不赢利的说法”,可以说明在墨盒cartridges的销售上取得了成功,选项中应该有cartridges

结论部分说的是这个分公司对于总公司的贡献下降~

削弱就可以从这里入手了,找一个原因使得总体贡献不会下降就可以了~

------------------------------------------------------------------------------------------------------------

GWD-7-Q37:

Escalating worldwide demand for corn has led to a sharp increase in the market price of corn, and corn prices are likely to remain high.  Corn is extensive used as feed for livestock, and because profit margins are tight in the livestock business, many farmers are expected to leave the business.  With fewer suppliers, meat prices will surely rise.  Nonetheless, observers expect an immediate short-term decrease in meat prices.

 

Which of the following, if true, most helps to justify the observers’ expectation?

 

  1. The increase in corn prices is due more to a decline in the supply of corn than to a growth in demand for it.

  2. Generally, farmers who are squeezed out of the livestock business send their livestock to market much earlier than they otherwise would.

  3. Some people who ate meat regularly in the past are converting to diets that include little or no meat.

  4. As meat prices rise, the number of livestock producers is likely to rise again.

  5. Livestock producers who stay in the business will start using feed other than corn more extensively than they did in the past.

结论应该和肉或是牲畜有关。B对的原因:那些被挤出生意的农民,为结束生意,通常没等livestock长到一定程度就卖掉(因为他们状况一样,被挤出市场时间差不多),从而导致市场短期供应过多而价格短期下降。

D可能会导致long term的猪肉价格下跌(但不保证),但绝不会导致 "immediate short-term decrease".

再次强调审题的重要性,审题就是审结论

------------------------------------------------------------------------------------------------------------

GWD-7-Q38:

Journalist:  Well-known businessman Arnold Bergeron has long been popular in the state, and he has often talked about running for governor, but he has never run.  However, we have just learned that Bergeron has fulfilled the financial disclosure requirement for candidacy by submitting a detailed list of his current financial holdings to the election commission.  So, it is very likely that Bergeron will be a candidate for governor this year.

 

The answer to which of the following questions would be most useful in evaluating the journalist’s argument?

 

  1. Has anybody else who has fulfilled the financial disclosure requirement for the upcoming election reported greater financial holdings than Bergeron?

  2. Is submitting a list of holdings the only way to fulfill the election commission’s financial disclosure requirements?

  3. Did the information recently obtained by the journalists come directly from the election commission?

  4. Have Bergeron’s financial holdings increased in value in recent years?

E.      Had Bergeron also fulfilled the financial disclosure requirements for candidacy before any previous gubernatorial elections?

   Answer: E

------------------------------------------------------------------------------------------------------------

GWD-7-Q41:

Magazine Publisher:  Our magazine does not have a liberal bias.  It is true that when a book review we had commissioned last year turned out to express distinctly conservative views, we did not publish it until we had also obtained a second review that took a strongly liberal position.  Clearly, however, our actions demonstrate not a bias in favor of liberal views but rather a commitment to a balanced presentation of diverse opinions.

 

Determining which of the following would be most useful in evaluating the cogency of the magazine publisher’s response?

 

  1. Whether any other magazines in which the book was reviewed carried more than one review of the book

  2. Whether the magazine publishes unsolicited book reviews as well as those that it has commissioned

  3. Whether in the event that a first review commissioned by the magazine takes a clearly liberal position the magazine would make any efforts to obtain further reviews

  4. Whether the book that was the subject of the two reviews was itself written from a clearly conservative or a clearly liberal point of view

  5. Whether most of the readers of the magazine regularly read the book reviews that the magazine publishes

观点:我们的杂志没有倾向自由派,是中立的。理由:去年我们委托的书评持保守派观点,我们没立即出版,而是等到持自由派观点的书评出来才一起出版。问题:评估原文ARGUMENT。原文理由中明显看出杂志倾向自由派,如果相反,持自由派观点的书评先出来,而杂志没先出版,而等持保守派观点的书评出来才一起出版,才能证明杂志没有倾向自由派。即C


[此贴子已经被作者于2007-5-17 15:59:01编辑过]
9#
 楼主| 发表于 2007-5-17 15:11:00 | 只看该作者

GWD-8-Q3:

In the late 1980’s, the population of sea otters in the North Pacific Ocean began to decline. Of the two plausible explanations for the decline—increased predation by killer whales or disease—disease is the more likely. After all, a concurrent sharp decline in the populations of seals and sea lions was almost certainly caused by a pollution-related disease, which could have spread to sea otters, whereas the population of killer whales did not change noticeably.

 

 

Which of the following, if true, most seriously weakens the reasoning?

 

 

A. Killer whales in the North Pacific usually prey on seals and sea lions but will, when this food source is scarce, seek out other prey.

B. There is no indication that substantial numbers of sea otters migrated to other locations from the North Pacific in the 1980’s.

C. Along the Pacific coast of North America in the 1980’s, sea otters were absent from many locations where they had been relatively common in former times.

D. Following the decline in the population of the sea otters, there was an increase in the population of sea urchins, which are sea otters’ main food source.

E. The North Pacific populations of seals and sea lions cover a wider geographic area than does the population of sea otters.

[A]

污染病导致海象和海狮(这两个鲸的主食)都死了,鲸就开始吃别了了(如海獭),提出另外一个造成sea otters 减少的可能原因,从而造成sea otters began to decline,削弱了作者的结论—disease is the more likely

其它标明的都是无关

------------------------------------------------------------------------------------------------------------

GWD-8-Q12:

A certain automaker aims to increase its market share by deeply discounting its vehicles’ prices for the next several months. The discounts will cut into profits, but because they will be heavily advertised the manufacturer hopes that they will attract buyers away from rival manufacturers’ cars. In the longer term, the automaker envisions that customers initially attracted by the discounts may become loyal customers.

 

 

In assessing the plan’s chances of achieving its aim, it would be most useful to know which of the following?

 

 

A. Whether the automaker’s competitors are likely to respond by offering deep discounts on their own products

B. Whether the advertisements will be created by the manufacturer’s current advertising agency

C. Whether some of the automaker’s models will be more deeply discounted than others

D. Whether the automaker will be able to cut costs sufficiently to maintain profit margins even when the discounts are in effect

E. Whether an alternative strategy might enable the automaker to enhance its profitability while holding a constant or diminishing share of the market

[A]文中提到了竞争者,所以竞争者是有关选项,不是无关。其它的广告方式,品牌细分,利润都无关

------------------------------------------------------------------------------------------------------------

GWD-8-Q13:

With a record number of new companies starting up in Derderia, and with previously established companies adding many new jobs, a record number of new jobs were created last year in the Derderian economy. This year, previously established companies will not be adding as many new jobs overall as such companies added last year. Therefore, unless a record number of companies start up this year, Derderia will not break its record for new jobs created.

 

 

Which of the following is an assumption on which the argument relies?

 

 

A. In a given year, new companies starting up create more new jobs on average than do previously established companies.

B. The number of people seeking employment is no larger this year than it was last year.

C. This year, the new companies starting up will not provide substantially more jobs per company than did new companies last year.

D. Previously established companies in Derderia will be less profitable this year than such companies were last year.

E. The number of jobs created in the Derderian economy last year was substantially larger than the number of jobs lost.

[C]主要在AC原文结论是新工作没去年多。新工作分两部分previously established companies 产生的新工作和new companies 产生的新工作。原文已知道previously established companies 产生的新工作没去年多,需要的假设是今年的new companies 产生的新工作和去年new companies 产生的新工作比较情况。对于A所说的new companies 产生的新工作比previously established companies 产生的新工作多和原文的结论没关系。比较对象错误。

new companies指代唯一

题干上说:除非今年新增加的公司数量上能有比去年多,否则总的工作机会就会比去年少。C去掉not后:今年每家公司创造的工作比去年要多-->那就有可能:即使数量少,但创造的工作机会反而多--->削弱!所以C

------------------------------------------------------------------------------------------------------------

GWD-8-Q22:

When storing Renaissance oil paintings, museums conform to standards that call for careful control of the surrounding temperature and humidity, with variations confined within narrow margins. Maintaining this environment is very costly, and recent research shows that even old oil paint is unaffected by wide fluctuations in temperature and humidity. Therefore, museums could relax their standards and save money without endangering their Renaissance oil paintings.

 

 

Which of the following is an assumption on which the argument depends?

 

 

A. Renaissance paintings were created in conditions involving far greater fluctuations in temperature and humidity than those permitted by current standards.

B. Under the current standards that museums use when storing Renaissance oil paintings, those paintings do not deteriorate at all.

C. Museum collections typically do not contain items that are more likely to be vulnerable to fluctuations in temperature and humidity than Renaissance oil paintings.

D. None of the materials in Renaissance oil paintings other than the paint are vulnerable enough to relatively wide fluctuations in temperature and humidity to cause damage to the paintings.

E. Most Renaissance oil paintings are stored in museums located in regions near the regions where the paintings were created.

[D]
  
Something is deteriorated do not mean it is endangered. So B is not a required condition to get the conclusion.

不选B的原因在于B过于绝对。

old oil painting old oil paint之间存在gap,答案就是如何弥补这个gap,应该是D

D是排除他因的假设,即没有其它颜料方面的导致画坏掉,取非,“画还是要因为颜料原因坏掉”和文章最后一句结论不符。Clikely再次不对C中的items that are more likely to be vulnerable to fluctuations in temperature and humidity than Renaissance oil paintings.
  
无关

------------------------------------------------------------------------------------------------------------

GWD-8-Q33:错了。

In order to withstand tidal currents, juvenile horseshoe crabs frequently burrow in the sand. Such burrowing discourages barnacles from clinging to their shells. When fully grown, however, the crabs can readily withstand tidal currents without burrowing, and thus they acquire substantial populations of barnacles. Surprisingly, in areas where tidal currents are very weak, juvenile horseshoe crabs are found not to have significant barnacle populations, even though they seldom burrow.

 

 

Which of the following, if true, most helps to explain the surprising finding?

 

 

A. Tidal currents do not themselves dislodge barnacles from the shells of horseshoe crabs.

B. Barnacles most readily attach themselves to horseshoe crabs in areas where tidal currents are weakest.

C. The strength of the tidal currents in a given location varies widely over the course of a day.

D. A very large barnacle population can significantly decrease the ability of a horseshoe crab to find food.

E. Until they are fully grown, horseshoe crabs shed their shells and grow new ones several times a year.

[E]不会。要先找到矛盾(或奇怪现象)。

奇怪现象:Such burrowing discourages barnacles from clinging to their shells即一般juvenile horseshoe crabs 会附着 barnacles 但是, juvenile horseshoe crabs are found not to have significant barnacle populations, even though they seldom burrow(这排除BURROW使它们没有附着barnacle的原因)。所以你要找另外原因解释 found not to have significant barnacle E的意思为 juvenile horseshoe crabs
   
Until they are fully grown指在他们长大前)一年有几次脱旧壳,长新壳。所以上面的barnacle都给脱掉了,当然没发现 barnacle

A是削弱,A和原文第二句话矛盾,Tidal currents 无法去掉 crabs barnaclesB与原文重复无新信息,C与地点无关。D无关。没看懂Eshed

还是要紧扣题目:是否能解释差异

------------------------------------------------------------------------------------------------------------

GWD-8-Q38:

Unless tiger hunting decreases, tigers will soon be extinct in the wild. The countries in which the tigers’ habitats are located are currently debating joint legislation that would ban tiger hunting. Thus, if these countries can successfully enforce this legislation, the survival of tigers in the wild will be ensured.

 

 

The reasoning in the argument is most vulnerable to criticism on the grounds that the argument

 

 

A. assumes without sufficient warrant that a ban on tiger hunting could be successfully enforced

B. considers the effects of hunting on tigers without also considering the effects of hunting on other endangered animal species

C. fails to take into account how often tiger hunters are unsuccessful in their attempts to kill tigers

D. neglects to consider the results of governmental attempts in the past to limit tiger hunting

E.
    
takes the removal of an impediment to the tigers’ survival as a guarantee of their survival

[E]
  
1.这是指出逻辑错误题,不是WEAKEN

2。原文推理:因为NOT EXTINCT----->HUNTING DECREASE。所以HUNTING DECREASBAN HUNTING成功)------>NOT EXTINCT。很容易看出逻辑错误为逆命题的错误,即将必要条件(HUNTING DECREASE)当作充分条件。即E

打猎会导致老虎灭绝,但不让打猎就不会导致老虎灭绝吗?不一定。

对于逻辑题,有一点很重要,就是答案不可能和原文冲突.如原文说已经执行,答案说没有执行,这个答案肯定不对.A

------------------------------------------------------------------------------------------------------------

GWD-8-Q40:

From 1973 to 1976, total United States consumption of cigarettes increased 3.4 percent, and total sales of chewing tobacco rose 18.0 percent. During the same period, total United States population increased 5.0 percent.

 

 

If the statements above are true, which of the following conclusions can be properly drawn?

 

 

A. United States manufacturers of tobacco products had higher profits in 1976 than in 1973.

B. Per capita consumption of cigarettes in the United States was lower in 1976 than in 1973.

C. The proportion of nonsmokers in the United States population dropped slightly between 1973 and 1976.
     

D. United States manufacturers of tobacco products realize a lower profit on cigarettes than on chewing tobacco.

E. A large percentage of United States smokers switched from cigarettes to chewing tobacco between 1973 and 1976.
     


    
答案当然是B。人口增长是5%consumption of cigarettes 增长才3.4%。当然人均consumption of cigarettes 变小了。

GWD-9-Q8:

Many large department stores in Montalia now provide shopping carts for their customers. Since customers using shopping carts tend to buy more than those without shopping carts, most of these stores are experiencing strong sales growth, while most other department stores are not. Therefore, in order to boost sales, managers of Jerrod’s, Montalia’s premier department store, are planning to purchase shopping carts and make them available to the store’s customers next month.

 

Which of the following, if true, casts most doubt whether the managers’ plan, if implemented, will achieve its goal?

 

A. Since most customers associate shopping carts with low-quality discount stores, Jerrod’s high-quality image would likely suffer if shopping carts were introduced.

B. Because the unemployment rate has declined to very low levels, Jerrod’s now has to pay significantly higher wages in order to retain its staff.

C. A number of department stores that did not make shopping carts available to their customers have had to close recently due to falling profits.

D. Shopping carts are not very expensive, but they generally need to be replaced every few years.

E. Stores that make shopping carts available to their customers usually have to hire people to retrieve the carts from parking areas.

[A] 理解答案A,关键透彻理解A的两层意思:1.J原来给人的印象是HIGH-QUALITY2.SHOPPING CART给人的印象是low-quality.这两者的矛盾让人合理的想到引进shopping cart会使J的顾客去J的购买欲望受损,从而影响sales,即WEAKEN结论  E错在它只影响利润,结论讲的是SALE无关

 A:嗯,他因削弱

B:与原文的reasoning完全无关

C:加强吧

Devery few years无关;注意结论的特殊性

E:只要购入shopping carts都会有这个问题,无法削弱

------------------------------------------------------------------------------------------------------------


[此贴子已经被作者于2007-5-17 16:03:02编辑过]
10#
 楼主| 发表于 2007-5-17 15:12:00 | 只看该作者

GWD-9-Q12:

A mosquito bite can transmit to a person the parasite that causes malaria, and the use of mosquito nets over children’s beds can significantly reduce the incidence of malarial infection for children in areas where malaria is common. Yet public health officials are reluctant to recommend the use of mosquito nets over children’s beds in such areas.

 

Which of the following, if true, would provide the strongest grounds for the public health officials’ reluctance?

 

A. Early exposure to malaria increases the body’s resistance to it and results in a lesser likelihood of severe life-threatening episodes of malaria.

B. Mosquito bites can transmit to people diseases other than malaria.

C. Mosquito nets provide protection from some insect pests other than mosquitoes.

D. Although there are vaccines available for many childhood diseases, no vaccine has been developed that is effective against malaria.

E. The pesticides that are most effective against mosquitoes in regions where malaria is common have significant detrimental effects on human health.

[A]

------------------------------------------------------------------------------------------------------------

GWD-9-Q14:
                
错了

The Eurasian ruffe, a fish species inadvertently不注意地 introduced into North America’s Great Lakes in recent years, feeds on the eggs of lake whitefish, a native species, thus threatening the lakes’ natural ecosystem. To help track the ruffe’s spread, government agencies have produced wallet-sized cards about the ruffe. The cards contain pictures of the ruffe and explain the danger they pose; the cards also request anglers to report any ruffe they catch.

 

Which of the following, if true, would provide most support for the prediction that the agencies’ action will have its intended effect?

 

A. The ruffe has spiny fins that make it unattractive as prey.

B. Ruffe generally feed at night, but most recreational fishing on the Great Lakes is done during daytime hours.

C. Most people who fish recreationally on the Great Lakes are interested in the preservation of the lake whitefish because it is a highly prized高奖赏的
            
game fish.
供垂钓的鱼

D. The ruffe is one of several nonnative species in the Great Lakes whose existence threatens the survival of lake whitefish populations there.

E. The bait that most people use when fishing for whitefish on the Great Lakes is not attractive to ruffe.

[C] A的意思为RUFFE不是好猎物。和原文预测无关。C说在G湖的钓鱼的多数人喜欢钓WHITEFISH。又因为RUFFEWHITEFISH的蛋为食,所以WHITEFISH多的地方RUFFE也一定多,这样钓到RUFFE的可能性就很大,跟踪RUFFE的行踪的目标更易达到,所以支持预测。

ruffe的确有害,但是政府现在的目的不是要灭了它,而是首先要track the ruffe's spread,政府不可能每天派一大笔人在湖边等着ruffe所以最有效的办法就是要人们参与进来积极举报鱼的行踪,所以政府作了很多小卡片让人们来带在身边,一是让人们了解这种鱼是什么样子,二是了解它有什么害处。但是这个计划的一个弱点就是如果人们都不配合,也就是拿了卡片就扔,看到ruffe也不通知政府,这个措施对于track the ruffe's spread的目的,显然是没有效果的!所以c加强说,因为人们有兴趣保护whitefish,那么当他们看到卡片说ruffe会残害whitefish的卡片会比较容易会配合政府来track ruffe。所以这个措施也就比较likely to have its intended effect==>to track the ruffe's spread.

BE是削弱直接看到Cpreservation of whitefish就可以选了

------------------------------------------------------------------------------------------------------------

GWD-9-Q16:错了

A diet high in saturated fats increases a person’s risk of developing heart disease. Regular consumption of red wine reduces that risk. Per-capita consumption of saturated fats is currently about the same in France as in the United States, but there is less heart disease there than in the United States because consumption of red wine is higher in France. The difference in regular red-wine consumption has been narrowing, but no similar convergence in heart-disease rates has occurred.
                   
convergence n.
集中, 收敛

 

Which of the following, if true, most helps to account for the lack of convergence noted above?

 

A. Consumption of saturated fats is related more strongly to the growth of fatty deposits on artery动脉 walls, which reduce blood flow to the heart, than it is to heart disease directly.

B. Over the past 30 years, per-capita consumption of saturated fats has remained essentially unchanged in the United States but has increased somewhat in France.

C. Reports of the health benefits of red wine have led many people in the United States to drink red wine regularly.与原文重复

D. Cigarette smoking, which can also contribute to heart disease, is only slightly more common in France than in the United States.无关

E. Regular consumption of red wine is declining dramatically among young adults in France, and heart disease typically does not manifest itself until middle age.

[E]饱和脂肪酸会增加心脏病的危险,但日常饮用红酒会减少这个危险。法国和美国人均对饱和脂肪酸的消费量是一样的,但法国的心脏病发病率低于美国,车为法国人喝更多的红酒。两国红酒的消费差距在减小(即美国赶上来了法国消费量少了。),但心脏病的发病率还是一样(即美国没有因为多喝红酒而减少心脏病发病率)——即法国年轻人少喝红酒(虽然美国人没有多喝),但病情要到中年才发作,而美国人的消费正常,发病率和原先也一样。

B所说,即法国的per-capita consumption of saturated fats 增加,则heart disease会增加,合美国的CONVERGENCE就有可能,与结论矛盾,解释不了lack of convergence noted

原文给两个规律:FATHEART DISEASE也高。WINEHEART DISEASE也低。

过去:法国(FAT一样,WINE高,HEART DISEASE低)

美国(FAT一样,WINE低,HEART DISEASE高)

现在:WINE的差别已缩小,却HEART DISEASE没缩小。

B说美国的FAT没变,法国的FAT增加点,既然法国的FAT增加,HEART DISEASE也应增加,它和美国在HEART DISEASE方面的差距应缩小但原文要你解释的是没缩小。

 ------------------------------------------------------------------------------------------------------------

GWD-9-Q20:

Which of the following most logically completes the reasoning?

 

Either food scarcity or excessive hunting can threaten a population of animals. If the group faces food scarcity, individuals in the group will reach reproductive maturity later than otherwise. If the group faces excessive hunting, individuals that reach reproductive maturity earlier will come to predominate. Therefore, it should be possible to determine whether prehistoric mastodons became extinct because of food scarcity or human hunting, since there are fossilized mastodon remains from both before and after mastodon populations declined, and ______.

 

A. there are more fossilized mastodon remains from the period before mastodon populations began to decline than from after that period与原文内容重复,无新信息,更多的化石也是这些信息。

B. the average age at which mastodons from a given period reached reproductive maturity can be established from their fossilized remains
            
和原文对应,原文讲的都是这两种因素对生育时间的影响。

C. it can be accurately estimated from fossilized remains when mastodons became extinct与灭绝时间无关

D. it is not known when humans first began hunting mastodons与人的狩猎无关

E. climate changes may have gradually reduced the food available to mastodons与气候无关


            
拉拉理解:和原文对应,原文讲的都是这两种因素对生育时间的影响。AB易混。

原文知两个关系:如果FACE FOOD SCARCITY,则慢到reproductive maturity 。如果FACE EXCESSIVE HUNTING,则早到reproductive maturity (早到reproductive maturity 的占大多数)。

所以作者认为有可能确定M是由于FOOD SCARCITY灭绝,还是EXCESSIVE HUNTING灭绝。理由是有M数量减少前后的化石。另外,B说可从化石知道到达reproductive maturity 的平均年龄。因为如果减少前的reproductive maturity 年龄大于减少后的年龄,则原因是EXCCESIVE HUNTING(根据第二个关系)。如果小于,则是FOOD SCARCITY(第一个关系)

原文内容+选项,构成答案

------------------------------------------------------------------------------------------------------------

GWD-9-Q24:

GWD-10-Q7:

For similar cars and drivers, automobile insurance for collision damage has always cost more in Greatport than in Fairmont. Police studies, however, show that cars owned by Greatport residents are, on average, slightly less likely to be involved in a collision than cars in Fairmont. Clearly, therefore, insurance companies are making a greater profit on collision-damage insurance in Greatport than in Fairmont.

 

Which of the following is an assumption on which the argument depends?

 

A. Repairing typical collision damage does not cost more in Greatport than in Fairmont.

B. There are no more motorists in Greatport than in Fairmont.

C. Greatport residents who have been in a collision are more likely to report it to their insurance company than Fairmont residents are.

D. Fairmont and Greatport are the cities with the highest collision-damage insurance rates.

E. The insurance companies were already aware of the difference in the likelihood of collisions before the publication of the police reports.

[A]

 

------------------------------------------------------------------------------------------------------------

GWD-9-Q28:错了

Editorial in Krenlandian Newspaper:

 

Krenland’s steelmakers are losing domestic sales because of lower-priced imports, in many cases because foreign governments subsidize their steel industries in ways that are banned by international treaties. But whatever the cause, the cost is ultimately going to be jobs in Krenland’s steel industry. Therefore, it would protect not only steel companies but also industrial employment in Krenland if our government took measures to reduce cheap steel imports.

 

Which of the following, if true, most seriously weakens the editorial’s argument?

 

A. Because steel from Krenland is rarely competitive in international markets, only a very small portion of Krenlandian steelmakers’ revenue comes from exports.与出口无关

B. The international treaties that some governments are violating by giving subsidies to steelmakers do not specify any penalties for such violations.

C. For many Krenlandian manufacturers who face severe international competition in both domestic and export markets, steel constitutes a significant part of their raw material costs.

D. Because of advances in order-taking, shipping, and inventory systems, the cost of shipping steel from foreign producers to Krenland has fallen considerably in recent years.说明成本更低,更需要保护,加强。

E. Wages paid to workers in the steel industry in Krenland differ significantly from wages paid to workers in many of the countries that export steel to Krenland.

[C]
            
原文推理:因为低价的进口钢原材挤掉了K地钢原材生产商的市场,使K地钢铁业失业增多。所以如果政府减少低价钢原材进口,就可减少全国的失业。

CK地钢原材价格构成了钢材加工品价格的主要部分。在这情况下,如果限制低价钢原材进口,就等于提高钢加工品价格,使钢材加工品竞争力减少,导致更多的失业(其它行业的总失业远大于钢铁行业的失业)。从而WEAKEN结论。

有两个GAP,一个是钢铁制造商和全国制造商,另一方面是钢铁行业的失业和全国总失业。

------------------------------------------------------------------------------------------------------------

GWD-9-Q30:
                        

Criminologist: Some legislators advocate mandating a sentence of life in prison for anyone who, having twice served sentences for serious crimes, is subsequently convicted of a third serious crime. These legislators argue that such a policy would reduce crime dramatically, since it would take people with a proven tendency to commit crimes off the streets permanently. What this reasoning overlooks, however, is that people old enough to have served two prison sentences for serious crimes rarely commit more than one subsequent crime. Filling our prisons with such individuals would have exactly the opposite of the desired effect, since it would limit our ability to incarcerate younger criminals, who commit a far greater proportion of serious crimes.

 

In the argument as a whole, the two boldfaced portions play which of the following roles?

 

A. The first is a conclusion that the argument as a whole seeks to refute; the second is a claim that has been advanced in support of that conclusion.

B. The first is a conclusion that the argument as a whole seeks to refute; the second is the main conclusion of the argument.

C. The first is the main conclusion of the argument; the second is an objection that has been raised against that conclusion.

D. The first is the main conclusion of the argument; the second is a prediction made on the basis of that conclusion.

E. The first is a generalization about the likely effect of a policy under consideration in the argument; the second points out a group of exceptional cases to which that generalization does not apply.


            
MAIN IDEA
就是该段文作者的主要意思,作者支持的。这篇短文的作者是CRIMINOLOGIST。他/她到底要支持什麽,表达什麽,不难看出吧。第一句BOLD可是LEGISTLATORS的意见,是CRIMINOLOGIST要反对的。

------------------------------------------------------------------------------------------------------------

GWD-9-Q37:

Researchers took a group of teenagers who had never smoked and for one year tracked whether they took up smoking and how their mental health changed. Those who began smoking within a month of the study’s start were four times as likely to be depressed at the study’s end than those who did not begin smoking. Since nicotine in cigarettes changes brain chemistry, perhaps thereby affecting mood, it is likely that smoking contributes to depression in teenagers.

 

Which of the following, if true, most strengthens the argument?

 

A.        Participants who were depressed at the study’s start were no more likely to be smokers at the study’s end than those who were not depressed.

B.        Participants who began smoking within a month of the study’s start were no more likely than those who began midway through to have quit smoking by the study’s end.

C.        Few, if any, of the participants in the study were friends or relatives of other participants.

D.       Some participants entered and emerged from a period of depression within the year of the study.

E.        The researchers did not track use of alcohol by the teenagers.

Answer:排除法较明显原文:调查开始后一个月内吸烟的人更容易忧郁。答案:不是忧郁才导致的吸烟。(A取非即:如果是由于忧郁导致的吸烟,就刚好变成了因果倒置的削弱)

本题答案排除了因果倒置,所以是加强。

要研究吸烟是否会导致忧郁症的实验显然是不可能以忧郁症患者为研究对象的,否则岂不是很容易导致原因和结果的混淆(即分不清到底是吸烟导致了忧郁还是忧郁导致了吸烟)。所以实验对象里面是不大可能有忧郁症患者的。A

------------------------------------------------------------------------------------------------------------

GWD-9-Q38:

GWD-12-Q21:

GWD-13-Q23:

In January of last year the Moviemania chain of movie theaters started propping its popcorn in canola oil, instead of the less healthful coconut oil that it had been using until then. Now Moviemania is planning to switch back, saying that the change has hurt popcorn sales. That claim is false, however, since according to Moviemania’s own sales figures, Moviemania sold 5 percent more popcorn last year than in the previous year.

 

Which of the following, if true, most strongly supports the argument against Moviemania’s claim?

 

A. Total sales of all refreshments at Moviemania’s movie theaters increased by less than 5 percent last year.

B. Moviemania makes more money on food and beverages sold at its theaters than it does on sales of movie tickets.

C. Moviemania’s customers prefer the taste of popcorn popped in coconut oil to that of popcorn popped in canola oil.

D. Total attendance at Moviemania’s movie theaters was more than 20 percent higher last year than the year before.

E. The year before last, Moviemania experienced a 10 percent increase in popcorn sales over the previous year.

Answer:

首先, the claim saying that the change has hurt popcorn sales.要反对这句话,才能符合题目意思.

题目中提出一个原因:

That claim is false, however, since according to moviemania’s own sales figures, moviemania sold 5percent more popcorn last year than in the previous year.

a答案提供了一个支持,提出是其他的refreshment销量减少了.

这说明moviemania的观点the change has hurt popcorn sales是不对的.A

------------------------------------------------------------------------------------------------------------

GWD-9-Q40:错了

Agricultural societies cannot exist without staple crops. Several food plants, such as kola and okra, are known to have been domesticated in western Africa, but they are all supplemental, not staple, foods. All the recorded staple crops grown in western Africa were introduced from elsewhere, beginning, at some unknown date, with rice and yams. Therefore, discovering when rice and yams were introduced into western Africa would establish the earliest date at which agricultural societies could have arisen there. 结论

 

Which of the following is an assumption on which the argument depends?

 

A. People in western Africa did not develop staple crops that they stopped cultivating once rice and yams were introduced.

B. There are no plants native to western Africa that, if domesticated, could serve as staple food crops.

C. Rice and yams were grown as staple crops by the earliest agricultural societies outside of western Africa.

D. Kola and okra are better suited to growing conditions in western Africa than domesticated rice and yams are.

E. Kola and okra were domesticated in western Africa before rice and yams were introduced there.

Answer: B 取非为There are some plants native to western Africa that, if domesticated, could serve as staple food crops

因为即使这样,你不知这些PLANTS domesticated是在rice and yams were introduced into western Africa
                        
之前还是之后,如果之后,则结论仍能成立。即B取非不一定使结论不成立。而A取非是结论一定不成立

 A


[此贴子已经被作者于2007-5-17 16:02:08编辑过]
您需要登录后才可以回帖 登录 | 立即注册

Mark一下! 看一下! 顶楼主! 感谢分享! 快速回复:

手机版|ChaseDream|GMT+8, 2024-4-26 08:46
京公网安备11010202008513号 京ICP证101109号 京ICP备12012021号

ChaseDream 论坛

© 2003-2023 ChaseDream.com. All Rights Reserved.

返回顶部